You are on page 1of 38

(1) PHILIPPINE SOCIETY FOR THE PREVENTION OF CRUELTY TO ANIMALS

vs.
COMMISSION ON AUDIT, ET. AL.,
GR. NO. 169752, September 25, 2007.
FACTS: Petitioner PSPCA (PHILIPPINE SOCIETY FOR THE PREVENTION OF CRUELTY TO
ANIMALS ) was incorporated as a juridical entity by virtue of Act No. 1285, enacted on
January 19, 1905, by the Philippine Commission. Act No. 1285 existed before both the
Corporation Law and the constitution of the Securities and Exchange Commission. The
objects of the petitioner, as stated in Section 2 of its charter, shall be to enforce laws relating
to cruelty inflicted upon animals or the protection of animals in the Philippine Islands, and
generally, to do and perform all things which may tend in any way to alleviate the suffering of
animals and promote their welfare.
Later, a COA Audit Team visited the Petitioner's office to have an audit survey pursuant to COA
Office Order No. 2003-051 dated November 18, 2003. Petitioner claimed that it was a private
entity not under the jurisdiction of COA. COA, through memorandums, maintained that
Petitioner is a government entity subject to audit jurisdiction. Petitioner received on September
27, 2005 the subject COA Office Order 2005-021 dated September 14, 2005 and the COA Letter
dated September 23, 2005. The Office Order and Letter reiterated audit survey. Hence,
Petitioner filed a petition for certiorari under Rule 65 of the ROC.
ISSUE: Whether Phil. Society for the Prevention of Cruelty to Animals is a government
entity.
HELD: NO. It is not government entity but a Quasi-Public Corporation. When a certain
juridical entity is impressed with public interest, it does not, by the circumstances alone,
make the entity a public corporation, inasmuch as a corporation may be private though
its charter contains provisions of a public character incorporated solely for public good.
To determine whether a corporation is public or private is found in the totality of the
relation of the corporation to the State.
A Quasi-public corporation is a specie of private corporations that renders
public service, supplies public want, and other charitable objectives. While purposely
organized for the gain and benefit of its members, they are required by law to discharge
functions for the public benefit. The qualifying factor is the type of service the former
renders to the public. If it performs public service, then it becomes a quasi-public
corporation.
If a corporation created by the State as the latter's own agency or instrumentality to
help it in carrying out its governmental functions, then that corporation is considered public.

violators of animal welfare laws and (2) share 50% of the fines imposed and collected through its
efforts pursuant to the violations of related laws.

However, Commonwealth Act No. 148 recalled the said powers. President Quezon then issued
Executive Order No. 63 directing the Commission of Public Safety, Provost Marshal General as
head of the Constabulary Division of the Philippine Army, Mayors of chartered cities and every
municipal president to detail and organize special officers to watch, capture, and prosecute
offenders of criminal-cruelty laws.

On December 1, 2003, an audit team from the Commission on Audit visited petitioners office to
conduct a survey. PSPCA demurred on the ground that it was a private entity and not under the
CoAs jurisdiction, citing Sec .2(1), Art. IX of the Constitution.

Issues:
WON the PSPCA is subject to CoAs Audit Authority.

Held:
No.
The charter test cannot be applied. It is predicated on the legal regime established by the 1935
Constitution, Sec.7, Art. XIII. Since the underpinnings of the charter test had been introduced by
the 1935 Constitution and not earlier, the test cannot be applied to PSPCA which was
incorporated on January 19, 1905. Laws, generally, have no retroactive effect unless the
contrary is provided. There are a few exceptions: (1) when expressly provided; (2) remedial
statutes; (3) curative statutes; and (4) laws interpreting others.

PSPCA vs Commission on Audit

None of the exceptions apply in the instant case.


Facts:
PSPCA was incorporated as a juridical entity by virtue of Act No. 1285 by the Philippine
Commission in order to enforce laws relating to the cruelty inflicted upon animals and for the
protection of and to perform all things which may tend to alleviate the suffering of animals and
promote their welfare.

In order to enhance its powers, PSPCA was initially imbued with (1) power to apprehend

The mere fact that a corporation has been created by a special law doesnt necessarily qualify it
as a public corporation. At the time PSPCA was formed, the Philippine Bill of 1902 was the
applicable law and no proscription similar to the charter test can be found therein. There was no
restriction on the legislature to create private corporations in 1903. The amendments introduced
by CA 148 made it clear that PSPCA was a private corporation, not a government agency.

PSPCAs charter shows that it is not subject to control or supervision by any agency of the State.
Like all private corporations, the successors of its members are determined voluntarily and
solely by the petitioner, and may exercise powers generally accorded to private corporations.

PSPCAs employees are registered and covered by the SSS at the latters initiative and not
through the GSIS.

The fact that a private corporation is impressed with public interest does not make the entity a
public corporation. They may be considered quasi-public corporations which areprivate
corporations that render public service, supply public wants and pursue other exemplary
objectives. The true criterion to determine whether a corporation is public or private is found in
the totality of the relation of the corporate to the State. It is public if it is created by the latters
own agency or instrumentality, otherwise, it is private.

(2) Boy Scouts of the Philippines


vs.
Commission on Audit (2011) [Vital Role of the Youth]
Petition: petition for prohibition with preliminary injunction and temporary restraining order
DOCTRINE:
An institution that molds and prepares the youth to become model citizens and outstanding
leaders of the country through lessons in patriotism, civic consciousness and moral values,
ultimately redounds to the benefit of public welfare and the state. The aforementioned functions
are undeniably sovereign functions enshrined under the Art. II- Sec. 13 of the Constitution
FACTS:
-The BSP is a public corporation created under Commonwealth Act No. 111 dated October 31,
1936, and whose functions relate to the fostering of public virtues of citizenship and patriotism
and the general improvement of the moral spirit and fiber of the youth.
-On Aug 19, 1999, COA issued Resolution No. 99-011 "Defining the Commission's policy with
respect to the audit of the Boy Scouts of the Philippines" which provides for the conduction of an
annual financial audit of the Boy Scouts of the Phil. and the expression of an opinion on the
fairness of their financial statements. The BSP shall also be classified among the government
corporations belonging to the Educational, Social, Scientific, Civic and Research Sector.

-On July 3, 2000, Director Sunico, Corporate Audit Officer of the COA, furnished the BSP with a
copy of the Memorandum that opined that the substantial government participation is only one
(1) of the three (3) grounds relied upon by the Court in the resolution of the case. Other
considerations include the character of the BSP's purposes and functions which has a public
aspect and the statutory designation of the BSP as a "public corporation". On the argument that
BSP is not "a government instrumentality" and "agency" of the government, the Supreme Court
has elucidated this matter in the BSP vs NLRC case when it declared that BSP is both a
"government-controlled corporation with an original charter" and as an "instrumentality" of the
Government.
-Upon the BSP's request, the audit was deferred for thirty (30) days. The BSP then filed a
Petition for Prohibition with Prayer for Preliminary Injunction and/or Temporary Restraining Order
before the COA.
ISSUES: W/N the BSP is a public corporation and is subject to COAs audit jurisdiction.
PROVISIONS:
-Commonwealth Act No. 111 (Boy Scout Charter), or An Act to Create a Public Corporation to
be Known as the Boy Scouts of the Philippines, and to Define its Powers and Purposes: Section
3.The purpose of this corporation shall be to promote, through organization, and cooperation
with other agencies, the ability of boys to do things for themselves and others, to train them in
scoutcraft, and to teach them patriotism, courage, self-reliance, and kindred virtues, using the
methods which are now in common use by boy scouts.
-Section 2(1), Article IX-D of the Constitution provides that COA shall have the power,
authority, and duty to examine, audit and settle all accounts pertaining to the revenue and
receipts of, and expenditures or uses of funds and property, owned or held in trust by, or
pertaining to, the Government, or any of its subdivisions, agencies or instrumentalities, including
government-owned or controlled corporations with original charters
-ART II- Section 13 of the Constitution. The State recognizes the vital role of the youth in
nation-building and shall promote and protect their physical, moral, spiritual, intellectual, and
social well-being. It shall inculcate in the youth patriotism and nationalism, and encourage their
involvement in public and civic affairs.
Article 44 of the Civil Code:
The following are juridical persons:
(1)The State and its political subdivisions;
(2)Other corporations, institutions and entities for public interest or purpose created by
law; their personality begins as soon as they have been constituted according to law;
(3)Corporations, partnerships and associations for private interest or purpose to which the law
grants a juridical personality, separate and distinct from that of each shareholder, partner or
member

- The COA resolution stated that the BSP was created as a public corporation under
Commonwealth Act No. 111 and is a government-controlled corporation. The COA Resolution
also cited its constitutional mandate under Section 2 (1), Article IX (D).

RULING + RATIO: Yes. BSP is a public corporation and its funds are subject to the COA's audit
jurisdiction.

-On Nov. 26, 1999, the BSP National President Jejomar Binay sought reconsideration of the
resolution stating that the BSP is not subject to the Commission's jurisdiction because it is not a
unit of the government. Moreover, RA 7278 virtually eliminated the "substantial government
participation" in the National Executive Board and that the BSP is not as a government
instrumentality under the 1987 Administrative Code which provides that instrumentality refers to
"any agency of the National Government, not integrated within the department framework,
vested with special functions or jurisdiction by law.

The BSP is a public corporation whose functions relate to the fostering of public virtues of
citizenship and patriotism and the general improvement of the moral spirit and fiber of the youth.
The functions of the BSP include, among others, the teaching to the youth of patriotism,
courage, self-reliance, and kindred virtues, are undeniably sovereign functions enshrined under
the Constitution. Any attempt to classify the BSP as a private corporation would be
incomprehensible since no less than the law which created it had designated it as a public
corporation and its statutory mandate embraces performance of sovereign functions. The
manner of creation and the purpose for which the BSP was created indubitably prove that it is a
government agency.

Moreover, there are three classes of juridical persons under Article 44 of the Civil Code and the
BSP, as presently constituted under Republic Act No. 7278, falls under the second
classification.
The purpose of the BSP as stated in its amended charter shows that it was created in order to
implement a State policy declared in Article II, Section 13 of the Constitution.
Evidently, the BSP, which was created by a special law to serve a public purpose in pursuit of a
constitutional mandate, comes within the class of "public corporations" defined by paragraph 2,
Article 44 of the Civil Code and governed by the law which creates it.

DISPOSITION: WHEREFORE, premises considered, the instant petition for prohibition is


DISMISSED.

Dissenting Opinion relevant to the issue: (Carpio)


According to Carpio, the public purpose of the BSP is not determinative of status. The
BSP performs functions which may be classified as public in character, in the sense that it
promotes "virtues of citizenship and patriotism and the general improvement of the moral spirit
and fiber of our youth." However, this fact alone does not automatically make the BSP a GOCC.
The fact that a certain juridical entity is impressed with public interest does not, by that
circumstance alone, make the entity a public corporation, incorporated solely for the
public good. Authorities are of the view that the purpose alone of the corporation cannot be
taken as a safe guide, for the fact is that almost all corporations are nowadays created to
promote the interest, good, or convenience of the public.
-The true criterion to determine whether a corporation is public or private is found in the totality
of the relation of the corporation to the State. If the corporation is created by the State as the
latter's own agency or instrumentality to help it in carrying out its governmental functions, then
that corporation is considered public; otherwise, it is private.

(3) ALHAMBRA CIGAR & CIGARETTE MANUFACTURING COMPANY, INC vs. SEC
G.R. No. L-23606
July 29, 1968
FACTS: Alhambra Cigar and Cigarette Manufacturing Company, Inc. was duly incorporated
under Philippine laws on January 15, 1912. By its corporate articles it was to exist for fifty (50)
years from incorporation. Its term of existence expired on January 15, 1962. On that date, it
ceased transacting business, entered into a state of liquidation. Thereafter, a new corporation,
Alhambra Industries, Inc., was formed to carry on the business of Alhambra. On June 20, 1963,
within Alhambra's three-year statutory period for liquidation, RA 3531 was enacted into law. It
amended Section 18 of the Corporation Law empowering domestic private corporations to
extend their corporate life beyond the period fixed by the articles of incorporation for a term not
to exceed fifty years in any one instance. Previous to RA 3531, the maximum non-extendible
term of such corporations was fifty years. On July 15, 1963, at a special meeting, Alhambra's
board of directors resolved to amend paragraph "Fourth" of its articles of incorporation to extend
its corporate life for an additional fifty years, or a total of 100 years from its incorporation.
Alhambra's articles of incorporation as so amended certified correct by its president and
secretary and a majority of its board of directors, were then filed with SEC. SEC, however,
returned said amended articles of incorporation to Alhambra's counsel with the ruling that RA
3531 "which took effect only on June 20, 1963, cannot be availed of by the said corporation, for
the reason that its term of existence had already expired when the said law took effect in short,
said law has no retroactive effect."
ISSUE: Whether or not a corporation can extend its life by amendment of its articles of
incorporation effected during the three-year statutory period for liquidation when its original term
of existence had already expired.
RULING: Plain from the language of the provision of Section 77 of Corporation Law is its
meaning: continuance of a "dissolved" corporation as a body corporate for three years has for its

purpose the final closure of its affairs, and no other; the corporation is specifically enjoined from
"continuing the business for which it was established". The liquidation of the corporation's affairs
set forth in Section 77 became necessary precisely because its life had ended. For this reason
alone, the corporate existence and juridical personality of that corporation to do business may
no longer be extended. And it should be clearly evident that no corporation in a state of
liquidation can act in any way, much less amend its articles, "for the purpose of continuing the
business for which it was established".

HELD: No. Alhambra cannot avail of the new law because it has already expired at the time of
its passage. When a corporation is liquidating pursuant to the statutory period of three years to
liquidate, it is only allowed to continue for the purpose of final closure of its business and no
other purposes. In fact, within that period, the corporation is enjoined from continuing the
business for which it was established. Hence, Alhambras board cannot validly amend its
articles of incorporation to extend its lifespan.

24 SCRA 269 Business Organization Corporation Law Corporate Lifespan


(5) PC Javier &Sons Inc. v. Court of Appeals
G.R. No. 129552. June 29, 2005
Chico- Nazario, J.:
Facts:
On January 15, 1912, Alhambra Cigar & Cigarette Manufacturing Company, Inc. was

PC Javier & Sons applied with First Summa Bank for a loan accommodation under the Industrial

incorporated. Its lifespan was for 50 years so on January 15, 1962, it expired. Thereafter, its

Guarantee Loan Fund (IGLF).

Board authorized its liquidation. Under the prevailing law, Alhambra has 3 years to liquidate.

The corporation through Pablo Javier was advised that its loan application was approved and
that the same shall be forwarded to the Central Bank for processing.

In 1963, while Alhambra was liquidating, Republic Act 3531 was enacted. It amended Section 18
of the Corporation Law; it empowered domestic private corporations to extend their corporate life

The CB released the loan.

beyond the period fixed by the articles of incorporation for a term not to exceed fifty years in any
one instance. Previous to Republic Act 3531, the maximum non-extendible term of such
corporations was fifty years.

To secure the loan, Javier executed CM over some machinery in favor of the bank. In the
meantime, the bank changed its name to PAIC Savings and Mortgage Bank Inc.
Thereafter, the corporation failed to pay; this prompted the Bank to move for the extrajudicial

Alhambra now amended its articles of incorporation to extend its lifespan for another 50 years.

foreclosure of the mortgages.

The Securities and Exchange Commission (SEC) denied the amended articles of incorporation.
PC Javier filed an action to restrain the extrajudicial foreclosure on the ground that it First
Summa and PAIC Bank are separate entities.

ISSUE: Whether or not a corporation under liquidation may still amend its articles of
incorporation to extend its lifespan.

Issue:
Whether the debtor should be formally notified of the corporate creditors change of name
Held:
NO. There is no such requirement under the law or any regulation ordering a bank that changes
its corporate name to formally notify all its debtors. This being the case, the court cannot impose

on the bank that changes its corporate name to notify its debtors of such change absent any
law, circular or regulation requiring it. Formal notification is therefore discretionary on the bank.

in the warrants, were actually seized; (3) the warrants were issued to fish evidence against the
aforementioned petitioners in deportation cases filed against them; (4) the searches and
seizures were made in an illegal manner; and (5) the documents, papers and cash money
seized were not delivered to the courts that issued the warrants, to be disposed of in accordance
with law.
In their answer, respondents-prosecutors alleged, (1) that the contested search warrants are
valid and have been issued in accordance with law; (2) that the defects of said warrants, if any,
were cured by petitioners' consent; and (3) that, in any event, the effects seized are admissible
in evidence against herein petitioners, regardless of the alleged illegality of the aforementioned
searches and seizures.
The documents, papers, and things seized under the alleged authority of the warrants in
question may be split into two (2) major groups, namely: (a) those found and seized in the
offices of the aforementioned corporations, and (b) those found and seized in the residences of
petitioners herein.
Issues/Held: WON as to the first group, petitioners as officers of the Corporation have a cause
of action to assail the legality of the contested warrants and of the seizures made in pursuance
thereof.- NO
Ratio:
Said corporations have their respective personalities, separate and
distinct from the personality of herein petitioners, regardless of the
amount of shares of stock or of the interest of each of them in said
corporations, and whatever the offices they hold therein may be.

(6) STONEHILL vs. DIOKNO


G.R. No. L-19550
June 19, 1967
Facts: A total of 42 search warrants against petitioners herein and/or the corporations of which
they were officers, directed to the any peace officer, to search the persons above-named and/or
the premises of their offices, warehouses and/or residences, and to seize and take possession
of the following personal property to wit:
Books of accounts, financial records, vouchers, correspondence, receipts, ledgers,
journals, portfolios, credit journals, typewriters, and other documents and/or papers
showing all business transactions including disbursements receipts, balance sheets
and profit and loss statements and Bobbins (cigarette wrappers).
as "the subject of the offense; stolen or embezzled and proceeds or fruits of the offense," or
"used or intended to be used as the means of committing the offense," which is described in the
applications adverted to above as "violation of Central Bank Laws, Tariff and Customs Laws,
Internal Revenue (Code) and the Revised Penal Code."
Petitioners allege that the search warrants are null and void because: (1) they do not describe
with particularity the documents, books and things to be seized; (2) cash money, not mentioned

It is well settled that the legality of a seizure can be contested only by the
party whose rights have been impaired thereby, and that the objection to an
unlawful search and seizure is purely personal and cannot be availed of by
third parties. If these papers were unlawfully seized and thereby the
constitutional rights of or any one were invaded, they were the rights of the
corporation and not the rights of the other defendants.
Consequently, petitioners herein may not validly object to the use in
evidence against them of the documents, papers and things seized from the
offices and premises of the corporations adverted to above, since the right
to object to the admission of said papers in evidence belongs exclusively to
the corporations, to whom the seized effects belong, and may not be
invoked by the corporate officers in proceedings against them in their
individual capacity.

(7) Bache and Co. v. Ruiz


GR No. L-32409
February 27, 1971
Facts:
In their petition Bache & Co. (Phil.), Inc., a corporation duly organized and existing under the
laws of the Philippines, and its President, Frederick E. Seggerman, pray this Court to: (1)
declare null and void the Search Warrant issued; (2) order respondents to desist from enforcing
the same and/or keeping the documents, papers and effects seized by virtue thereof, as well as
from enforcing the tax assessments on petitioner corporation alleged by petitioners to have been
made on the basis of the said documents, papers and effects; and (3) order the return of the
latter to petitioners.
At that time the request for the issuance of the Search Warrant was made by respondent De
Leon, with his witness Logronio, respondent Judge was hearing a certain case; so, by means of
a note, he instructed his Deputy Clerk of Court to take the depositions of respondents De Leon
and Logronio. After the session had adjourned, respondent Judge was informed that the
depositions had already been taken. The stenographer, upon request of respondent Judge, read
to him her stenographic notes; and thereafter, respondent Judge asked respondent Logronio to
take the oath and warned him that if his deposition was found to be false and without legal basis,
he could be charged for perjury. Respondent Judge signed respondent de Leon's application for
search warrant and respondent Logronio's deposition, the Search Warrant was then sign by
respondent Judge and accordingly issued.
Three days later, the BIR agents served the search warrant petitioners at the offices of petitioner
corporation on Ayala Avenue, Makati, Rizal. Petitioners' lawyers protested the search on the
ground that no formal complaint or transcript of testimony was attached to the warrant. The
agents nevertheless proceeded with their search which yielded six boxes of documents.
Documents were uses as basis in assessing the Corporation for tax deficiencies.
Issue/Held: WON a corporation is entitled to protection against unreasonable search and
seizures.- YES.
Ratio:
"Although, for the reasons above stated, we are of the opinion that an officer of a corporation
which is charged with a violation of a statute of the state of its creation, or of an act of Congress
passed in the exercise of its constitutional powers, cannot refuse to produce the books and
papers of such corporation, we do not wish to be understood as holding that a corporation is not
entitled to immunity, under the 4th Amendment, against unreasonable searches and seizures. A
corporation is, after all, but an association of individuals under an assumed name and with a
distinct legal entity. In organizing itself as a collective body it waives no constitutional immunities
appropriate to such body. Its property cannot be taken without compensation. It can only be
proceeded against by due process of law, and is protected, under the 14th Amendment, against
unlawful discrimination . . ." (Hale v. Henkel, 201 U.S. 43, 50 L. ed. 652.)
"In Linn v. United States, 163 C.C.A. 470, 251 Fed. 476, 480, it was thought that a different rule
applied to a corporation, the ground that it was not privileged from producing its books and
papers. But the rights of a corporation against unlawful search and seizure are to be protected
even if the same result might have been achieved in a lawful way." (Silverthorne Lumber
Company, et al. v. United States of America, 251 U.S. 385, 64 L. ed. 319.)
In Stonehill, et al. vs. Diokno, et al., supra, this Court impliedly recognized the right of a
corporation to object against unreasonable searches and seizures, thus:
"As regards the first group, we hold that petitioners herein have no cause of

action to assail the legality of the contested warrants and of the seizures
made in pursuance thereof, for the simple reason that said corporations have
their respective personalities, separate and distinct from the personality of
herein petitioners, regardless of the amount of shares of stock or the interest
of each of them in said corporations, whatever, the offices they hold therein
may be. Indeed, it is well settled that the legality of a seizure can be
contested only by the party whose rights have been impaired thereby, and
that the objection to an unlawful search and seizure is purely personal and
cannot be availed of by third parties. Consequently, petitioners herein may
not validly object to the use in evidence against them of the documents,
papers and things seized from the offices and premises of the corporations
adverted to above, since the right to object to the admission of said papers in
evidence belongs exclusively to the corporations, to whom the seized effects
belong, and may not be invoked by the corporate officers in proceedings
against them in their individual capacity . . ."
In the Stonehill case only the officers of the various corporations in whose offices documents,
papers and effects were searched and seized were the petitioners. In the case at bar, the
corporation to whom the seized documents belong, and whose rights have thereby been
impaired, is itself a petitioner.
Issue/Held: WON the Search Warrant is null and void.- YES
Ratio:
a) Respondent Judge failed to personally examine the complainant and his witness; his
participation was limited to listening to the stenographer's readings of her notes, to a
few words of warning against the commission of perjury, and to administering the oath
to the complainant and his witness. This cannot be considered a personal
examination.
b) The search warrant was issued for more than one specific offense. Search Warrant
was issued for "[v]iolation of Sec. 46(a) of the National Internal Revenue Code in
relation to all other pertinent provisions thereof particularly Secs. 53, 72, 73, 208 and
209."
c) The search warrant does not particularly describe the things to be seized.
BARREDO, J., concurring:
Search Warrant is null and void. The search warrant was issued for more than one specific
offense.

(8) BATAAN SHIPYARD AND ENGINEERING vs. PCGG


G.R. No. 75885 May 27, 1987
Facts:
Challenged by a private corporation known as the Bataan Shipyard and Engineering Co., Inc.
are: (1) Executive Orders Numbered 1 and 2, promulgated by President Corazon C. Aquino on
February 28, 1986 and March 12, 1986, respectively, and (2) the sequestration, takeover, and
other orders issued, and acts done, in accordance with said executive orders by the Presidential
Commission on Good Government and/or its Commissioners and agents, affecting said
corporation.
BASECO describes itself in its petition as "a shiprepair and shipbuilding company * *
incorporated as a domestic private corporation * * (on Aug. 30, 1972) by a consortium of Filipino
shipowners and shipping executives. Its main office is at Engineer Island, Port Area, Manila,
where its Engineer Island Shipyard is housed, and its main shipyard is located at Mariveles
Bataan." Barely six months after its incorporation, BASECO acquired from National Shipyard &
Steel Corporation, or NASSCO, a government-owned or controlled corporation, the latter's
shipyard at Mariveles, Bataan, known as the Bataan National Shipyard (BNS), and except for

NASSCO's Engineer Island Shops and certain equipment of the BNS, consigned for future
negotiation all its structures, buildings, shops, quarters, houses, plants, equipment and
facilities, in stock or in transit. This it did in virtue of a "Contract of Purchase and Sale with
Chattel Mortgage" executed on February 13, 1973. The price was P52,000,000.00.
Unaccountably, the price of P52,000,000.00 was reduced by more than one-half, to
P24,311,550.00, about eight (8) months later. A document to this effect was executed on
October 9, 1973, entitled "Memorandum Agreement," and was signed for NASSCO by Arturo
Pacificador, as Presiding Officer of the Board of Directors, and David R. Ines, as General
Manager. This agreement bore, at the top right corner of the first page, the word "APPROVED"
in the handwriting of President Marcos, followed by his usual full signature.
On October 1, 1974, BASECO acquired three hundred (300) hectares of land in Mariveles from
the Export Processing Zone Authority for the price of P10,047,940.00 of which, as set out in the
document of sale, P2,000.000.00 was paid upon its execution, and the balance stipulated to be
payable in installments.
Some nine months afterwards, or on July 15, 1975, to be precise, BASECO, again with the
intervention of President Marcos, acquired ownership of the rest of the assets of NASSCO which
had not been included in the first two (2) purchase documents. Transferred to BASECO were
NASSCO's "ownership and all its titles, rights and interests over all equipment and facilities
including structures, buildings, shops, quarters, houses, plants and expendable or semiexpendable assets, located at the Engineer Island, known as the Engineer Island Shops,
including all the equipment of the Bataan National Shipyards (BNS) which were excluded from
the sale of NBS to BASECO but retained by BASECO and all other selected equipment and
machineries of NASSCO at J. Panganiban Smelting Plant."
Other evidence submitted to the Court by the Solicitor General proves that President
Marcos not only exercised control over BASECO, but also that he actually owns well nigh
one hundred percent of its outstanding stock.
The Solicitor General has drawn the Court's attention to the intriguing circumstance that found in
Malacanang shortly after the sudden flight of President Marcos, were certificates corresponding
to more than ninety-five percent (95%) of all the outstanding shares of stock of BASECO,
endorsed in blank, together with deeds of assignment of practically all the outstanding shares of
stock of the three (3) corporations above mentioned (which hold 95.82% of all BASECO stock),
signed by the owners thereof although not notarized.
The Sequestration, Takeover, and Other Orders Complained of:
a.

b.

Basic sequestration order of various companies and The TAKEOVER


Order- While BASECO concedes that "sequestration without resorting to
judicial action, might be made within the context of Executive Orders Nos. 1
and 2 before March 25, 1986 when the Freedom Constitution was
promulgated, under the principle that the law promulgated by the ruler under
a revolutionary regime is the law of the land, it ceased to be acceptable
when the same ruler opted to promulgate the Freedom Constitution on
March 25, 1986 wherein under Section I of the same, Article IV (Bill of
Rights) of the 1973 Constitution was adopted providing, among others, that
"No person shall be deprived of life, liberty and property without due
process of law." (Const., Art. I V, Sec. 1)."
Order of production of business documents and records- BASECO argues
that the order to produce corporate records from 1973 to 1986, which it has
apparently already complied with, was issued without court authority and

infringed its constitutional right against self-incrimination, and unreasonable


search and seizure.
c. BASECO contends that the PCGG had unduly interfered with its right of
dominion and management of its business affairs on the following matters:
i. Orders Re Engineer Island1. Termination of Contract for Security Services
2. Change of Mode of Payment of Entry Charges
ii. Aborted contract for improvement of wharf at Engineer Island
iii. Order for Operation of Sesiman Rock Quarry, Mariveles, Bataan
iv. Order to Dispose of Scrap, etc.
v. Termination of Services of BASECO Officers
Executive Order No. 1
Executive Order No. 1 stresses the "urgent need to recover all ill-gotten wealth," and postulates
that "vast resources of the government have been amassed by former President Ferdinand E.
Marcos, his immediate family, relatives, and close associates both here and abroad." Upon
these premises, the Presidential Commission on Good Government was created, "charged with
the task of assisting the President in regard to (certain specified) matters," among which was
precisely* * The recovery of all in-gotten wealth accumulated by former President
Ferdinand E. Marcos, his immediate family, relatives, subordinates and
close associates, whether located in the Philippines or abroad, including the
takeover or sequestration of all business enterprises and entities owned or
controlled by them, during his administration, directly or through nominees,
by taking undue advantage of their public office and/or using their powers,
authority, influence, connections or relationship.
Executive Order No. 2
Executive Order No. 2 gives additional and more specific data and directions respecting "the
recovery of ill-gotten properties amassed by the leaders and supporters of the previous regime."
It declares that:
1) * * the Government of the Philippines is in possession of evidence
showing that there are assets and properties purportedly pertaining to
former Ferdinand E. Marcos, and/or his wife Mrs. Imelda Romualdez
Marcos, their close relatives, subordinates, business associates, dummies,
agents or nominees which had been or were acquired by them directly or
indirectly, through or as a result of the improper or illegal use of funds or
properties owned by the government of the Philippines or any of its
branches, instrumentalities, enterprises, banks or financial institutions, or by
taking undue advantage of their office, authority, influence, connections or
relationship, resulting in their unjust enrichment and causing grave damage
and prejudice to the Filipino people and the Republic of the Philippines:"
and
2) * * said assets and properties are in the form of bank accounts, deposits,
trust accounts, shares of stocks, buildings, shopping centers,
condominiums, mansions, residences, estates, and other kinds of real and
personal properties in the Philippines and in various countries of the world."
Executive Order No. 14
PCGG is empowered, "with the assistance of the Office of the Solicitor General and other
government agencies, * * to file and prosecute all cases investigated by it * * as may be
warranted by its findings." All such cases, whether civil or criminal, are to be filed "with the

Sandiganbayan which shall have exclusive and original jurisdiction thereof."

Issue/Held: WON the issuance of the sequestration and take-over orders was valid- Yes
Ratio: In the light of the affirmative showing by the Government that, prima facie at least, the
stockholders and directors of BASECO as of April, 1986 were mere "dummies," nominees or
alter egos of President Marcos; at any rate, that they are no longer owners of any shares of
stock in the corporation, the conclusion cannot be avoided that said stockholders and directors
have no basis and no standing whatever to cause the filing and prosecution of the instant
proceeding; and to grant relief to BASECO, as prayed for in the petition, would in effect be to
restore the assets, properties and business sequestered and taken over by the PCGG to
persons who are "dummies," nominees or alter egos of the former president.
The facts herein stated at some length do indeed show that the private corporation known as
BASECO was "owned or controlled by former President Ferdinand E. Marcos * * during his
administration, * * through nominees, by taking advantage of * * (his) public office and/or using *
* (his) powers, authority, influence * *," and that NASSCO and other property of the government
had been taken over by BASECO; and the situation justified the sequestration as well as the
provisional takeover of the corporation in the public interest, in accordance with the terms of
Executive Orders No. 1 and 2, pending the filing of the requisite actions with the Sandiganbayan
to cause divestment of title thereto from Marcos, and its adjudication in favor of the Republic
pursuant to Executive Order No. 14.
Issue/Held: WON the Executive Orders are Bills of Attainder.- NO, Executive Orders not a
Bill of Attainder.
Ratio:
1.

2.

Nothing in the executive orders can be reasonably construed as a determination or


declaration of guilt. On the contrary, the executive orders, inclusive of Executive Order
No. 14, make it perfectly clear that any judgment of guilt in the amassing or acquisition
of "ill-gotten wealth" is to be handed down by a judicial tribunal, in this case, the
Sandiganbayan, upon complaint filed and prosecuted by the PCGG.
N o punishment is inflicted by the executive orders, as the merest glance at their
provisions will immediately make apparent. In no sense, therefore, may the executive
orders be regarded as a bill of attainder.

Issue/Held: WON there is a violation of right against self-incrimination and unreasonable


searches and seizure.- NO
Ratio:
It is elementary that the right against self-incrimination has no application to juridical persons.
While an individual may lawfully refuse to answer incriminating questions
unless protected by an immunity statute, it does not follow that a
corporation, vested with special privileges and franchises, may refuse to
show its hand when charged with an abuse of such privileges
Relevant jurisprudence is also cited by the Solicitor General.
* * corporations are not entitled to all of the constitutional protections which
private individuals have. * * They are not at all within the privilege against
self-incrimination, although this court more than once has said that the
privilege runs very closely with the 4th Amendment's Search and Seizure
provisions. It is also settled that an officer of the company cannot refuse to
produce its records in its possession upon the plea that they will either
incriminate him or may incriminate it." (Oklahoma Press Publishing Co. v.

Walling, 327 U.S. 186; emphasis, the Solicitor General's).


* * The corporation is a creature of the state. It is presumed to be
incorporated for the benefit of the public. It received certain special
privileges and franchises, and holds them subject to the laws of the state
and the limitations of its charter. Its powers are limited by law. It can make
no contract not authorized by its charter. Its rights to act as a corporation
are only preserved to it so long as it obeys the laws of its creation. There is
a reserve right in the legislature to investigate its contracts and find out
whether it has exceeded its powers. It would be a strange anomaly to hold
that a state, having chartered a corporation to make use of certain
franchises, could not, in the exercise of sovereignty, inquire how these
franchises had been employed, and whether they had been abused, and
demand the production of the corporate books and papers for that purpose.
The defense amounts to this, that an officer of the corporation which is
charged with a criminal violation of the statute may plead the criminality of
such corporation as a refusal to produce its books. To state this proposition
is to answer it. While an individual may lawfully refuse to answer
incriminating questions unless protected by an immunity statute, it does not
follow that a corporation, vested with special privileges and franchises may
refuse to show its hand when charged with an abuse of such privileges.
(Wilson v. United States, 55 Law Ed., 771, 780 [emphasis, the Solicitor
General's])
At any rate, Executive Order No. 14-A, amending Section 4 of Executive Order No. 14 assures
protection to individuals required to produce evidence before the PCGG against any possible
violation of his right against self-incrimination. It gives them immunity from prosecution on the
basis of testimony or information he is compelled to present. As amended, said Section 4 now
provides that
The witness may not refuse to comply with the order on the basis of his
privilege against self-incrimination; but no testimony or other information
compelled under the order (or any information directly or indirectly derived
from such testimony, or other information) may be used against the witness
in any criminal case, except a prosecution for perjury, giving a false
statement, or otherwise failing to comply with the order.
The constitutional safeguard against unreasonable searches and seizures finds no application to
the case at bar either. There has been no search undertaken by any agent or representative of
the PCGG, and of course no seizure on the occasion thereof.

Issue: Whether the Magsaysay sister, allegedly stockholders of SUBIC, are interested parties in
a case where corporate properties are in dispute.
Held: Viewed in the light of Section 2, Rule 12 of the Revised Rules of Court, the Magsaysay
sisters have no legal interest in the subject matter in litigation so as to entitle them to intervene
in the proceedings. To be permitted to intervene in a pending action, the party must have a legal
interest in the matter in litigation, or in the success of either of the parties or an interest against
both, or he must be so situated as to be adversely affected by a distribution or other disposition
of the property in the custody of the court or an officer thereof. Here, the interest, if it exists at all,
of the Magsaysay sisters is indirect, contingent, remote, conjectural, consequential and
collateral. At the very least, their interest is purely inchoate, or in sheer expectancy of a right in
the management of the corporation and to share in the profits thereof and in the properties and
assets thereof on dissolution, after payment of the corporate debts and obligations. While a
share of stock represents a proportionate or aliquot interest in the property of the corporation, it
does not vest the owner thereof with any legal right or title to any of the property, his interest in
the corporate property being equitable or beneficial in nature. Shareholders are in no legal
sense the owners of corporate property, which is owned by the corporation as a distinct legal
person.

(9) Magsaysay-Labrador, et. al. vs. Court of Appeals [GR 58168, 19 December 1989]
Third Division, Fernan (CJ): 3 concur, 1 on leave
Facts: On 9 February 1979, Adelaida Rodriguez-Magsaysay, widow and special administratix of
the estate of the late Senator Genaro Magsaysay, brought before the then Court of First
Instance of Olongapo an action against Artemio Panganiban, Subic Land Corporation (SUBIC),
Filipinas Manufacturer's Bank (FILMANBANK) and the Register of Deeds of Zambales, for the
annulment of the Deed of Assignment executed by the late Senator in favor of SUBIC (as a
result of which TCT 3258 was cancelled and TCT 22431 issued in the name of SUBIC), for the
annulment of the Deed of Mortgage executed by SUBIC in favor of FILMANBANK (dated 28
April 1977 in the amount of P 2,700,000.00), and cancellation of TCT 22431 by the Register of
Deeds, and for the latter to issue a new title in her favor.
On 7 March 1979, Concepcion Magsaysay-Labrador, Soledad Magsaysay-Cabrera, Luisa
Magsaysay-Corpuz, Felicidad Magsaysay, and Mercedes Magsaysay-Diaz, sisters of the late
senator, filed a motion for intervention on the ground that on 20 June 1978, their brother
conveyed to them 1/2 of his shareholdings in SUBIC or a total of 416,566.6 shares and as
assignees of around 41 % of the total outstanding shares of such stocks of SUBIC, they have a
substantial and legal interest in the subject matter of litigation and that they have a legal interest
in the success of the suit with respect to SUBIC. On 26 July 1979, the trial court denied the
motion for intervention, and ruled that petitioners have no legal interest whatsoever in the matter
in litigation and their being alleged assignees or transferees of certain shares in SUBIC cannot
legally entitle them to intervene because SUBIC has a personality separate and distinct from its
stockholders.
On appeal, the Court of Appeals found no factual or legal justification to disturb the findings of
the lower court. The appellate court further stated that whatever claims the Magsaysay sisters
have against the late Senator or against SUBIC for that matter can be ventilated in a separate
proceeding. The motion for reconsideration of the Magsaysay sisters was denied. Hence, the
petition for review on certiorari.

MAGSAYSAY-LABRADOR, petitioner
vs.
COURT OF APPEALS, respondent
G.R. No. 58168 December 19, 1989
FACTS:
On February 9, 1979, Adelaida Rodriguez-Magsaysay, widow and special
administratix of the estate of the late Senator Genaro Magsaysay, brought before the then Court
of First Instance of Olongapo an action against Artemio Panganiban, Subic Land Corporation
(SUBIC), Filipinas Manufacturer's Bank (FILMANBANK) and the Register of Deeds of Zambales.
In her complaint, she alleged that in 1958, she and her husband acquired, thru conjugal funds, a
parcel of land with improvements, known as "Pequena Island", covered by TCT No. 3258; that
after the death of her husband, she discovered [a] an annotation at the back of TCT No. 3258
that "the land was acquired by her husband from his separate capital;" [b] the registration of a
Deed of Assignment dated June 25, 1976 purportedly executed by the late Senator in favor of
SUBIC, as a result of which TCT No. 3258 was cancelled and TCT No. 22431 issued in the
name of SUBIC; and [c] the registration of Deed of Mortgage dated April 28, 1977 in the amount
of P 2,700,000.00 executed by SUBIC in favor of FILMANBANK; that the foregoing acts were
void and done in an attempt to defraud the conjugal partnership considering that the land is
conjugal, her marital consent to the annotation on TCT No. 3258 was not obtained, the change
made by the Register of Deeds of the titleholders was effected without the approval of the
Commissioner of Land Registration and that the late Senator did not execute the purported
Deed of Assignment or his consent thereto, if obtained, was secured by mistake, violence and
intimidation. She further alleged that the assignment in favor of SUBIC was without
consideration and consequently null and void. She prayed that the Deed of Assignment and the
Deed of Mortgage be annulled and that the Register of Deeds be ordered to cancel TCT No.
22431 and to issue a new title in her favor.

(Intervest, for brevity) which the latter maintained with the defendant bank under account No.
0200-02027-8;

ISSUE:
Whether or not petitioners ownership in the outstanding capital stock of SUBIC
entitles them to a significant vote in the corporate affairs.
RULING:
NO.
The words "an interest in the subject" mean a direct interest in the cause of action as
pleaded, and which would put the intervenor in a legal position to litigate a fact alleged in the
complaint, without the establishment of which plaintiff could not recover.
Here, the interest, if it exists at all, of petitioners-movants is indirect, contingent,
remote, conjectural, consequential and collateral. At the very least, their interest is purely
inchoate, or in sheer expectancy of a right in the management of the corporation and to share in
the profits thereof and in the properties and assets thereof on dissolution, after payment of the
corporate debts and obligations.
While a share of stock represents a proportionate or aliquot interest in the property of
the corporation, it does not vest the owner thereof with any legal right or title to any of the
property, his interest in the corporate property being equitable or beneficial in nature.
Shareholders are in no legal sense the owners of corporate property, which is owned by the
corporation as a distinct legal person.

6. Although the checks were clearly payable to the plaintiff corporation and crossed on their face
and for payees account only, defendant bank accepted the checks to be deposited to the current
account of Intervest and thereafter presented the same for collection from the drawee bank
which subsequently cleared the same thus allowing Intervest to make use of the funds to the
prejudice of the plaintiff;
xxx
14. The plaintiff has demanded upon the defendant to restitute the amount representing the
value of the checks but defendant refused and continue to refuse to honor plaintiffs demands up
to the present;
15. As a result of the illegal and irregular acts perpetrated by the defendant bank, the plaintiff
was damaged to the extent of the amount of P31,663.88.
Premium prayed that judgment be rendered ordering defendant bank to pay the amount
of P31,663.88 representing the value of the checks plus interest, P100,000.00 as exemplary
damages; and P30,000.00 as attorneys fees.
In its Answer International Corporate Bank alleged, inter alia, that Premium has no
capacity/personality/authority to sue in this instance and the complaint should, therefore, be
dismissed for failure to state a cause of action.

(10) [G.R. No. 96551. November 4, 1996]


PREMIUM MARBLE RESOURCES, INC., petitioner, vs. THE COURT OF APPEALS and
INTERNATIONAL CORPORATE BANK, respondents.
PRINTLINE CORPORATION, petitioner, vs. THE COURT OF APPEALS and
INTERNATIONAL CORPORATE BANK, respondents.
DECISION
Assailed in the instant petition for review is the decision [1] of the Court of Appeals in CAG.R. CV No. 16810 dated September 28, 1990 which affirmed the trial courts dismissal of
petitioners complaint for damages.
The antecedents:
On July 18, 1986, Premium Marble Resources, Inc. (Premium for brevity), assisted by Atty.
Arnulfo Dumadag as counsel, filed an action for damages against International Corporate Bank
which was docketed as Civil Case No. 14413. The complaint states, inter alia:
3. Sometime in August to October 1982, Ayala Investment and Development Corporation issued
three (3) checks [Nos. 097088, 097414 & 27884] in the aggregate amount of P31,663.88
payable to the plaintiff and drawn against Citibank;
xxx
5. On or about August to October 1982, former officers of the plaintiff corporation headed by
Saturnino G. Belen, Jr., without any authority whatsoever from the plaintiff deposited the abovementioned checks to the current account of his conduit corporation, Intervest Merchant Finance

A few days after Premium filed the said case, Printline Corporation, a sister company of
Premium also filed an action for damages against International Corporate Bank docketed as
Civil Case No. 14444. Thereafter, both civil cases were consolidated.
Meantime, the same corporation, i.e., Premium, but this time represented by Siguion
Reyna, Montecillio and Ongsiako Law Office as counsel, filed a motion to dismiss on the ground
that the filing of the case was without authority from its duly constituted board of directors as
shown by the excerpt of the minutes of the Premiums board of directors meeting. [2]
In its opposition to the motion to dismiss, Premium thru Atty. Dumadag contended that the
persons who signed the board resolution namely Belen, Jr., Nograles & Reyes, are not directors
of the corporation and were allegedly former officers and stockholders of Premium who were
dismissed for various irregularities and fraudulent acts; that Siguion Reyna Law office is the
lawyer of Belen and Nograles and not of Premium and that the Articles of Incorporation of
Premium shows that Belen, Nograles and Reyes are not majority stockholders.
On the other hand, Siguion Reyna Law firm as counsel of Premium in a rejoinder, asserted
that it is the general information sheet filed with the Securities and Exchange Commission,
among others, that is the best evidence that would show who are the stockholders of a
corporation and not the Articles of Incorporation since the latter does not keep track of the many
changes that take place after new stockholders subscribe to corporate shares of stocks.
In the interim, defendant bank filed a manifestation that it is adopting in toto Premiums
motion to dismiss and, therefore, joins it in praying for the dismissal of the present case on the
ground that Premium lacks authority from its duly constituted board of directors to institute the
action.
In its Order, the lower court concluded that:
Considering that the officers (directors) of plaintiff corporation enumerated in the Articles of

Incorporation, filed on November 9, 1979, were to serve until their successors are elected and
qualified and considering further that as of March 4, 1981, the officers of the plaintiff corporation
were Alberto Nograles, Fernando Hilario, Augusto Galace, Jose L.R. Reyes, Pido Aguilar and
Saturnino Belen, Jr., who presumably are the officers represented by the Siguion Reyna Law
Firm, and that together with the defendants, they are moving for the dismissal of the aboveentitled case, the Court finds that the officers represented by Atty. Dumadag do not as yet have
the legal capacity to sue for and in behalf of the plaintiff corporation and/or the filing of the
present action (Civil Case 14413) by them before Case No. 2688 of the SEC could be decided is
a premature exercise of authority or assumption of legal capacity for and in behalf of plaintiff
corporation.
The issues raised in Civil Case No. 14444 are similar to those raised in Civil Case No. 14413.
This Court is of the opinion that before SEC Case No. 2688 could be decided, neither the set of
officers represented by Atty. Dumadag nor that set represented by the Siguion Reyna, Montecillo
and Ongsiako Law Office, may prosecute cases in the name of the plaintiff corporation.

We find the petition without merit.


The only issue in this case is whether or not the filing of the case for damages against
private respondent was authorized by a duly constituted Board of Directors of the petitioner
corporation.
Petitioner, through the first set of officers, viz., Mario Zavalla, Oscar Gan, Lionel Pengson,
Jose Ma. Silva, Aderito Yujuico and Rodolfo Millare, presented the Minutes[5] of the meeting of its
Board of Directors held on April 1, 1982, as proof that the filing of the case against private
respondent was authorized by the Board. On the other hand, the second set of officers, viz.,
Saturnino G. Belen, Jr., Alberto C. Nograles and Jose L.R. Reyes, presented a
Resolution[6] dated July 30, 1986, to show that Premium did not authorize the filing in its behalf of
any suit against the private respondent International Corporate Bank.
Later on, petitioner submitted its Articles of Incorporation [7] dated November 6, 1979 with
the following as Directors: Mario C. Zavalla, Pedro C. Celso, Oscar B. Gan, Lionel Pengson, and
Jose Ma. Silva.

It is clear from the pleadings filed by the parties in these two cases that the existence of a cause
of action against the defendants is dependent upon the resolution of the case involving intracorporate controversy still pending before the SEC.[3]

However, it appears from the general information sheet and the Certification issued by the
SEC on August 19, 1986[8] that as of March 4, 1981, the officers and members of the board of
directors of the Premium Marble Resources, Inc. were:

On appeal, the Court of Appeals affirmed the trial courts Order [4] which dismissed the
consolidated cases. Hence, this petition.

Alberto C. Nograles President/Director

Petitioner submits the following assignment of errors:


I

Fernando D. Hilario Vice President/Director


Augusto I. Galace Treasurer

The Court of Appeals erred in giving due course to the motion to dismiss filed by the
Siguion Reyna Law Office when the said motion is clearly filed not in behalf of the
petitioner but in behalf of the group of Belen who are the clients of the said law office.

Jose L.R. Reyes Secretary/Director


Pido E. Aguilar Director

II
Saturnino G. Belen, Jr. Chairman of the Board.
The Court of Appeals erred in giving due course to the motion to dismiss filed by the
Siguion Reyna Law Office in behalf of petitioner when the said law office had already
appeared in other cases wherein the petitioner is the adverse party.
III
The Court of Appeals erred when it ruled that undersigned counsel was not authorized by
the Board of Directors to file Civil Case Nos. 14413 and 14444.
IV
The Court of Appeals erred in concluding that under SEC Case No. 2688 the incumbent
directors could not act for and in behalf of the corporation.
V
The Court of Appeals is without jurisdiction to prohibit the incumbent Board of Directors
from acting and filing this case when the SEC where SEC Case No. 2688 is pending has
not even made the prohibition.

While the Minutes of the Meeting of the Board on April 1, 1982 states that the newly
elected officers for the year 1982 were Oscar Gan, Mario Zavalla, Aderito Yujuico and Rodolfo
Millare, petitioner failed to show proof that this election was reported to the SEC. In fact, the last
entry in their General Information Sheet with the SEC, as of 1986 appears to be the set of
officers elected in March 1981.
We agree with the finding of public respondent Court of Appeals, that in the absence of
any board resolution from its board of directors the [sic] authority to act for and in behalf of the
corporation, the present action must necessarily fail. The power of the corporation to sue and be
sued in any court is lodged with the board of directors that exercises its corporate powers. Thus,
the issue of authority and the invalidity of plaintiff-appellants subscription which is still pending, is
a matter that is also addressed, considering the premises, to the sound judgment of the
Securities & Exchange Commission.[9]
By the express mandate of the Corporation Code (Section 26), all corporations duly
organized pursuant thereto are required to submit within the period therein stated (30 days) to
the Securities and Exchange Commission the names, nationalities and residences of the
directors, trustees and officers elected.
Sec. 26 of the Corporation Code provides, thus:

Sec. 26. Report of election of directors, trustees and officers. Within thirty (30) days after the
election of the directors, trustees and officers of the corporation, the secretary, or any other
officer of the corporation, shall submit to the Securities and Exchange Commission, the names,
nationalities and residences of the directors, trustees and officers elected. xxx
Evidently, the objective sought to be achieved by Section 26 is to give the public
information, under sanction of oath of responsible officers, of the nature of business, financial
condition and operational status of the company together with information on its key officers or
managers so that those dealing with it and those who intend to do business with it may know or
have the means of knowing facts concerning the corporations financial resources and business
responsibility.[10]
The claim, therefore, of petitioners as represented by Atty. Dumadag, that Zaballa, et al.,
are the incumbent officers of Premium has not been fully substantiated. In the absence of an
authority from the board of directors, no person, not even the officers of the corporation, can
validly bind the corporation.[11]
We find no reversible error in the decision sought to be reviewed.
ACCORDINGLY, for lack of merit, the petition is hereby DENIED.

(10) PREMIUM MARBLE RESOURCES, INC.


vs.
THE COURT OF APPEALS and INTERNATIONAL CORPORATE BANK
1996 Nov 4, G.R. No. 96551
FACTS:
Herein petitioner filed a case for damages against respondent for allowing clearance
of checks by unauthorized officers of the former, to the formers prejudice. However this case
was opposed by some members of the petitioner on the ground that the filing of the complaint
was not authorized by the Board. Hence, a resolution of this case was necessary to litigate the
claim of the petitioner for damages against the respondent bank.
ISSUE:
Whether or not the filing was authorized by a duly constituted Board of Directors of the
petitioner corporation.
RULING:
NO.
The petitioners asserted that the Board authorized such filing. However, from the
records of the case as well as that of the corporations, no evidence was seen and shown that
the results of the election where the supposed members of the Board who allegedly authorized
the filing were filed with the Securities and Exchange Commission. The Corporation Code
mandates that within thirty (30) days after the election of the directors, trustees and officers of
the corporation, the secretary, or any other officer of the corporation, shall submit to the
Securities and Exchange Commission, the names, nationalities and residences of the directors,
trustees and officers elected. Failure to comply with such requirement, the elected members

cannot be considered as the duly constituted and elected members of the Board. Hence, being
not duly constituted, the filing of the case was not authorized by the Board.

(11) MONFORT HERMANOS AGRICULTURAL DEVELOPMENT CORPORATION vs


ANTONIO B. MONFORT III

Resolution was issued, does not automatically make the four signatories (i.e., Paul M.
Monfort, Yvete M. Benedicto, Jaqueline M. Yusay and Ester S. Monfort) to the said
Board Resolution (whose name do not appear in the 1996 General Information Sheet)
as among the incumbent Members of the Board. This is because it was not
established that they were duly elected to replace the said deceased Board Members.

G.R. No. 152542 : July 8, 2004


G.R. No. 155472 : July 8, 2004
Facts:
Monfort Hermanos Agricultural Development Corporation, a domestic private corporation, is the
registered owner of a farm, fishpond and sugar cane plantation known as Haciendas San
Antonio II, Marapara, Pinanoag and Tinampa-an, all situated in Cadiz City. It also owns one unit
of motor vehicle and two units of tractors. The same allowed Ramon H. Monfort, its Executive
Vice President, to breed and maintain fighting cocks in his personal capacity at Hacienda San
Antonio. In 1997, the group of Antonio Monfort III, through force and intimidation, allegedly took
possession of the 4 Haciendas, the produce thereon and the motor vehicle and tractors, as well
as the fighting cocks of Ramon H. Monfort.
In G.R. No. 155472: The Corporation, represented by its President, Ma. Antonia M. Salvatierra,
and Ramon H. Monfort, in his personal capacity, filed against the group of Antonio Monfort III, a
complaint for delivery of motor vehicle, tractors and 378 fighting cocks, with prayer for injunction
and damages. Motion to dismiss on the ground of Ma. Antonia M. Salvatierra's lack of capacity
to sue on behalf of the Corporation was denied.
In G.R. No. 152542: Ma. Antonia M. Salvatierra filed on behalf of the Corporation a complaint for
forcible entry, preliminary mandatory injunction with temporary restraining order and damages
against the group of Antonio Monfort III.
The group of Antonio Monfort III alleged that they are possessing and controlling the Haciendas
and harvesting the produce therein on behalf of the corporation and not for themselves. They
likewise raised the affirmative defense of lack of legal capacity of Ma. Antonia M. Salvatierra to
sue on behalf of the Corporation.
Complaint was eventually dismissed.
Basis of claim of Salvatierra\s lack of capacity to sue: The group of Antonio Monfort III claims
that the March 31, 1997 Board Resolution authorizing Ma. Antonia M. Salvatierra and/or Ramon
H. Monfort to represent the Corporation is void because the purported Members of the Board
who passed the same were not validly elected officers of the Corporation.
Issue/ Held: WON Ma. Antonia M. Salvatierra has the legal capacity to sue on behalf of the
Corporation. -NO. Ma. Antonia M. Salvatierra failed to prove that four of those who
authorized her to represent the Corporation were the lawfully elected Members of the
Board of the Corporation. As such, they cannot confer valid authority for her to sue on
behalf of the corporation.
Ratio:
A corporation has no power except those expressly conferred on it by the Corporation
Code and those that are implied or incidental to its existence. In turn, a corporation
exercises said powers through its board of directors and/or its duly authorized officers
and agents. Thus, it has been observed that the power of a corporation to sue and be
sued in any court is lodged with the board of directors that exercises its corporate
powers. In turn, physical acts of the corporation, like the signing of documents, can be
performed only by natural persons duly authorized for the purpose by corporate bylaws or by a specific act of the board of directors.
Corporation failed to comply with Section 26 of the Corporation Code, requiring submission to
the SEC within thirty (30) days after the election the names, nationalities and residences of the
elected directors, trustees and officers of the Corporation.
1.

In the case at bar, the fact that four of the six Members of the Board listed in the 1996
General Information Sheet are already dead at the time the March 31, 1997 Board

To correct the alleged error in the General Information Sheet, the retained accountant of the
Corporation informed the SEC in its November 11, 1998 letter that the non-inclusion of the
lawfully elected directors in the 1996 General Information Sheet was attributable to its oversight
and not the fault of the Corporation. This belated attempt, however, did not erase the doubt as to
whether an election was indeed held.
2.

What further militates against the purported election of those who signed the March
31, 1997 Board Resolution was the belated submission of the alleged Minutes of the
October 16, 1996 meeting where the questioned officers were elected. The issue of
legal capacity of Ma. Antonia M. Salvatierra was raised before the lower court by the
group of Antonio Monfort III as early as 1997, but the Minutes of said October 16,
1996 meeting was presented by the Corporation only in its September 29,
1999 Comment before the Court of Appeals. Moreover, the Corporation failed to prove
that the same October 16, 1996 Minutes was submitted to the SEC.

by Estela Lombos and Anita Pascual who allege that they are the duly authorized
representatives of petitioner corporation, without showing any proof whatsoever of
such authority.

The petitioner filed a Motion for Reconsideration arguing that that there was no
showing that the persons acting on its behalf were not authorized to do so and that its petition
was filed within the additional 15-day period granted by the CA. Attached to the Motion was a
Secretary's Certificate showing that petitioner's Board of Directors approved a Resolution on
February 11, 2000 appointing Estela Lombos and Anita Pascual, incumbent directors of the
corporation, as its duly authorized representatives who may sign all papers, execute all
documents, and do such other acts as may be necessary to prosecute the petition for review
that it would file with the CA

(12) PASCUAL & SANTOS v MEMBERS OF TRAMO LAKAS NEIGHBORHOOD ASSN


442 SCRA 448, GR No 144880, November 17, 2004

Facts:
The Members of Tramo Wakas Neighborhood Association, represented by Dominga
Magno (respondents), lodged before the Presidential Action Center a petition praying that
ownership over 3 parcels of land situated in Barangay San Dionisio, Paraaque, Metro Manila
with an aggregate area of 35,195 square meters be awarded to them. Respondents alleged that
petitioner claims ownership of the subject lots which they have openly, peacefully and
continuously occupied since 1957. This was referred to the Land Management Bureau (LMB).
The LMB found for respondents.

The appeal lodged by the petitioner before the Office of DENR Secretary was
dismissed for lack of merit. Both the LMB and the Office of the President affirmed the decision of
DENR.

Petitioner filed its petition for review with the CA and asked that the decision of the OP and
DENR be set aside and to declare that the subject lots as no longer forming part of the public
domain and have been validly acquired by petitioner. The CA dismissed the appeal due to infirm
Verification and Certification of non-forum shopping and belated filing.

For one, the Verification and Certification of non-forum shopping was signed merely

The CA again denied petitioners motion for reconsideration stressing that any person
who claims authority to sign, in behalf of another, the Certificate of Non-Forum Shopping, as
required by the rules, must show sufficient proof thereof. Bare allegations are not proof, and the
representation of one who acts in behalf of another cannot, by itself, serve as proof of his
authority to act as agent or of the extent of his authority as agent.

Issue: Whether or not the persons who executed the verification and certification of non-forum
shopping attached to PSIs manifestation/petition for review filed with the CA were authorized to
do so.

Ruling: Lombos and Pascual were authorized to execute the verification and certification of nonforum shopping. Case is remanded to the appellate court to give due course to the appeal.

When the petition for certiorari was filed with the CA, there was no proof attached
thereto that Lombos and Pascual were authorized to sign the verification and non-forum
shopping certification. Subsequent to the CA's dismissal of the petition, however, petitioner filed
a motion for reconsideration to which it attached a certificate issued by its board secretary
stating that on February 11, 2000 or prior to the filing of the petition, Lombos and Pascual had
been authorized by petitioner's board of directors to file the petition before the CA. It has been
ruled that the subsequent submission of proof of authority to act on behalf of a petitioner
corporation justifies the relaxation of the Rules for the purpose of allowing its petition to be given
due course.

And while the requirement of the certificate of non-forum shopping is mandatory,


nonetheless the requirements must not be interpreted too literally and thus defeat the objective
of preventing the undesirable practice of forum shopping.

The requirement that the petitioner should sign the certificate of non-forum shopping
applies even to corporations, considering that the mandatory directives of the Rules of Court
make no distinction between natural and juridical persons.

Except for the powers which are expressly conferred on it by the Corporation Code and those
that are implied by or are incidental to its existence, a corporation has no powers. It exercises its
powers through its board of directors and/or its duly authorized officers and agents. Thus, its
power to sue and be sued in any court is lodged with the board of directors that exercises its
corporate powers. Physical acts, like the signing of documents, can be performed only by
natural persons duly authorized for the purpose by corporate by-laws or by a specific act of the
board of directors.

(13) G.R. No. 150959 August 4, 2006


UNITED PARAGON MINING CORPORATION, Petitioner,
vs.
COURT OF APPEALS, former 12th DIVISION, ATTY. MURLY P. MENDEZ and CESARIO 1 F.
ERMITA,Respondents.
DECISION
GARCIA, J.:
Assailed and sought to be set aside in this petition for review under Rule 45 of the Rules of
Court is the Decision 2dated July 24, 2001 of the Court of Appeals (CA), as reiterated in its
Resolution 3 of November 7, 2001, dismissing the petition for certiorari with prayer for a
temporary restraining order and preliminary injunction thereat filed by the herein petitioner in CAG.R. SP No. 44450, entitled United Paragon Mining Corporation, represented by Feliciano M.
Daniel v. Atty. Murly P. Mendez, in his capacity as Accredited Voluntary Arbitrator, Region V, and
Cesario F. Ermita.
The facts:
Prior to the instant controversy, private respondent Cesario F. Ermita (Cesario, for brevity) was a
regular employee working as a foreman of petitioner United Paragon Mining Corporation
(UPMC, hereafter).
On January 18, 1996, Cesario received a termination letter bearing date January 16, 1996 and
signed by UPMCs Personnel Superintendent, Feliciano M. Daniel, informing Cesario that his
employment as foreman is terminated effective thirty days after his receipt of the letter. As stated
in the letter, the termination was on account of Cesarios violation of company rules against
infliction of bodily injuries on a co-employee, it being alleged therein that Cesario inflicted bodily
injuries on a co-employee, a certain Jerry Romero, as well as for unlawfully possessing a deadly
weapon, a bolo, again in violation of company rules.
As a result of the termination, the matter was brought to the grievance machinery as mandated
under the Collective Bargaining Agreement existing at that time between UPMC and the United
Paragon Supervisors Union. Having failed to reach a settlement thereat, the parties agreed to
submit the dispute to voluntary arbitration. Accordingly, the complaint for illegal dismissal was
referred to Voluntary Arbitrator Atty. Murly P. Mendez of the National Conciliation and Mediation
Board, Regional Branch No. V, Legaspi City, whereat the same was docketed as VA Case No.
RB5-657-04-002-96.
On February 28, 1997, Voluntary Arbitrator Mendez rendered a decision 4 in Cesarios favor,
stating that although the procedural requirements in the termination of an employee had been
complied with, the termination of Cesario was unjustified because it was arrived at through gross
misapprehension of facts. Explains the Voluntary Arbitrator:
An analysis of the tenor of the termination letter would seem to indicate that Ceasario Ermita
was separated from service simply because his explanation was not acceptable to the company.
Stated more bluntly, Ermita was terminated not because there was a definite finding of fact
relative to his supposed culpability, but because his answer did not find favor with management.
xxx xxx xxx

The evidence on record partakes of the uncorroborated statement of Jerry Romero claiming that
he was assaulted by [Cesario]. This claim has been disputed and is denied by [Cesario] in the
statement executed by him on January 2, 1996 as well as in his written explanation (Annex 6,
Respondent's Position Paper).
On this point, it can be argued that since this is a case of one's word against another, the best
that could be said of management's evidence is that it has achieved a level at an equi-poise with
that of the Constitution. The spirit of prevailing jurisprudence as well as a liberal interpretation of
the new Constitutional provision on labor, would mandate that where a doubt exists, the same
should be resolved in favor of labor. The position of [Cesario] appears to have been
strengthened by the document jointly signed by [him] and Jerry Romero, the supposed victim of
the assault charged.
This amicable settlement would serve to negate the charge of physical injury against [Cesario]
as a basis for termination, it appearing that even [his] supposed victim, Jerry Romero, who has
been made to appear as a complainant in the proceedings which resulted in the termination
letter, has admitted in this amicable settlement (Annex A, Complainant's Position Paper) that
"hindi naming sinasadya yon at itong ginawa naming sulat na ito ay siya ang magpapatunay na
ayos kaming dalawa at walang problema sa isa't isa."
This admission, that comes no less from the supposed accuser of [Cesario], clearly establishes
the fact that whatever may have happened between them on New Year's eve was something
that neither of them willfully and voluntarily did. Since it has been established that the supposed
scuffle between [Cesario] and Romero was "hindi sinasadya," then it would necessarily follow
that there could not have been a willful and voluntary assault by [Cesario] upon Romero. This
situation is further rendered more puzzling by the fact that the suspected assailant was himself
the bearer of the tell-tale marks of injury.
xxx xxx xxx
It has been established to the satisfaction of this Arbitrator that the bolo seen that night was
used to chop wood to be burnt in the bonfire. This statement by people who happened to be
unbiased and disinterested remains uncontested and undisputed.
Further, the preponderance of evidence shows that it was not [Cesario] who used said bolo, but
his son.
xxx xxx xxx
On these points, it is the finding of this Arbitrator, and it is so ruled, that Ceasario Ermita was
unjustifiably terminated. 5 (Words in brackets supplied).
On the basis of the above, the Voluntary Arbitrator, in his aforementioned decision of February
28, 1997, ordered Cesarios reinstatement, to wit:
WHEREFORE, judgment is hereby issued ordering respondent United Paragon Mining
Corporation to immediately reinstate Ceasario F. Ermita to his former position prior to the
termination without loss of seniority nor interruption of service, and to pay said Ceasario F.
Ermita his back wages, including such other fringe benefits as he would have been entitled to,
from the date of his termination effective February 17, 1996 up to the time of actual
reinstatement. Attorney's fees are hereby granted equivalent to 10 per cent of such monetary
award as the complainant is entitled to.

For lack of merit, all other claims for damages are hereby dismissed.
SO ORDERED.
In time, UPMC moved for a reconsideration of the decision insofar as it ordered Cesarios
reinstatement which UPMC sought to avert by offering separation pay instead. UPMC cites the
following against the decreed reinstatement: 1) Cesarios position has already been filled up;
and 2) reinstatement is no longer appropriate in view of the supposed strained relations between
Cesario and UPMC.
In his Order 6 of April 22, 1997, the Voluntary Arbitrator denied the desired reconsideration
stressing that UPMCs management misapprehended the facts when it caused Cesarios
termination, which cannot support the claim of the existence of strained relations between him
and the corporation.
Unsatisfied, UPMC, thru its Personnel Superintendent Feliciano M. Daniel, elevated the case to
the CA on a Petition for Certiorari with Prayer for Temporary Restraining Order and Injunction,
thereat docketed as CA-G.R. SP No. 44450, asserting that the Voluntary Arbitrator committed
grave abuse of discretion, erroneous interpretation of the law and denial of substantial justice.
In the herein assailed Decision 7 dated July 24, 2001, the CA, without going into the merits of the
petition, dismissed the same on the following grounds:
1) The petition for certiorari was not the proper remedy in order to seek review or nullify
decisions or final orders issued by the Labor Arbiter;
2) The verification in the petition is ineffective and insufficient because it was merely signed by
the company's Personnel Superintendent without alleging or showing that he is authorized for
the said purpose and that the verification was based on knowledge and information;
3) The petitioner's ground of grave abuse of discretion, erroneous interpretation of the law and
denial of justice are actually dwelling on the appreciation of facts, which cannot be entertained in
a petition for certiorari.
With its motion for reconsideration having been denied by the CA in its Resolution of November
7, 2001, 8petitioner UPMC is now with this Court via the present recourse, submitting for our
consideration the following questions:
I
WHETHER OR NOT THE COURT OF APPEALS ERRED IN DISMISSING THE PETITION
AFTER FINDING THAT THE PROPER REMEDY SHOULD HAVE BEEN A PETITION FOR
REVIEW ON CERTIORARI AND NOT A PETITION FOR CERTIORARI;
II
WHETHER OR NOT THE PUBLIC RESPONDENT COURT OF APPEALS ERRED IN
DISMISSING THE PETITION AFTER FINDING THAT THE VERIFICATION PORTION OF THE
PETITION WAS INEFFECTIVE AND INSUFFICIENT IN THE ABSENCE OF ALLEGATION OR
SHOWING THAT FELICIANO DANIEL, AS PERSONNEL SUPERINTENDENT WAS DULY
AUTHORIZED TO FILE THE PETITION;

III

Securities and Exchange Commission."

WHETHER OR NOT THE PUBLIC RESPONDENT COURT OF APPEALS ERRED IN


DISMISSING THE PETITION AFTER FINDING THAT THE PETITION LACKS MERIT
BECAUSE IT DWELLED ON THE APPRECIATION OF FACTS WHICH IS NOT PROPER IN
PETITION FOR CERTIORARI.

Given the reality that the petition in CA-G.R. SP No. 44450 was filed by Daniel in behalf of and in
representation of petitioner UPMC without an enabling resolution of the latters board of
directors, that petition was fatally defective, inclusive of the verification and the certification of
non-forum shopping executed by Daniel himself.

The recourse must have to be DENIED, no reversible error having been committed by the CA in
its challenged decision.

True, ample jurisprudence exists to the effect that subsequent and substantial compliance of a
petitioner may call for the relaxation of the rules of procedure in the interest of justice. 11 But to
merit the Court's liberal consideration, petitioner must show reasonable cause justifying noncompliance with the rules and must convince the Court that the outright dismissal of the petition
would defeat the administration of justice. 12 Here, petitioner has not adequately explained its
failure to have the certification against forum shopping signed by its duly authorized officer.
Instead, it merely persisted in its thesis that it was not necessary to show proof that its
Personnel Superintendent was duly authorized to file that petition and to sign the verification
thereof and the certification against forumshopping despite the absence of the necessary board
authorization, thereby repeating in the process its basic submission that CA-G.R. SP No. 44450
is merely a continuation of the proceedings before the Voluntary Arbitrator and that its Personnel
Superintendent was impleaded as one of the respondents in Cesarios complaint for illegal
dismissal.

We start with the basic concept that a corporation, like petitioner UPMC, has no power except
those expressly conferred on it by the Corporation Code and those that are implied or incidental
to its existence. In turn, a corporation exercises said powers through its board of directors and/or
its duly authorized officers and agents. It has thus been observed that the power of a corporation
to sue and be sued in any court is lodged with its board of directors that exercises its corporate
powers. In turn, physical acts of the corporation, like the signing of documents, can be
performed only by natural persons duly authorized for the purpose by the corporate by-laws or
by a specific act of the board of directors. 9
It is petitioners posture that there is no necessity for a board resolution authorizing its Personnel
Superintendent to file in its behalf the certiorari petition in CA-G.R. SP No. 44450 because said
petition arose out of the labor dispute filed against it and its Personnel Superintendent, Feliciano
M. Daniel. It is argued that in Cesarios complaint for illegal dismissal, Daniel was made a corespondent of the corporation. Upon this premise, UPMC argues that Daniel has all the right to
answer the complaint and to appeal an unfavorable judgment therein, which he actually did, in
his capacity as the corporations Personnel Superintendent and as its representative. Plodding
on, petitioner contends that were the CA to insist that Daniel could not represent the corporation,
it follows that the proceedings before the Voluntary Arbitrator could only be binding as against
Daniel because the company then could not have been duly represented in said proceedings.
Throughout the proceedings before the Voluntary Arbitrator, that is, from the filing of the position
papers up to the filing of the motion for reconsideration, UPMC was duly represented by its
counsel, Atty. Archimedes O. Yanto. True it is that Cesarios complaint for illegal dismissal was
filed against the corporation and Daniel. It appears obvious to us, however, that Daniel was
merely a nominal party in that proceedings, as in fact he was impleaded thereat in his capacity
as UPMCs Personnel Superintendent who signed the termination letter. For sure, Cesarios
complaint contains no allegation whatsoever for specific claim or charge against Daniel in
whatever capacity. As it is, Daniel was not in anyway affected by the outcome of the illegal
dismissal case because only the corporation was made liable therein to Cesario. Being not a
real party-in-interest, Daniel has no right to file the petition in CA-G.R. SP No. 44450 in behalf of
the corporation without any authority from its board of directors. It is basic in law that a
corporation has a legal personality entirely separate and distinct from that of its officers and the
latter cannot act for and on its behalf without being so authorized by its governing board.
In Premium Marble Resources, Inc. v. Court of Appeals, 10 we made it clear that in the absence
of an authority from the board of directors, no person, not even the officers of the corporation,
can validly bind the latter:
We agree with the finding of public respondent Court of Appeals, that "in the absence of any
board resolution from its board of directors the [sic] authority to act for and in behalf of the
corporation, the present action must necessary fail. The power of the corporation to sue and be
sued in any court is lodged with the board of directors that exercises its corporate powers. Thus,
the issue of authority and the invalidity of plaintiff-appellants subscription which is still pending,
is a matter that is also addressed, considering the premises, to the sound judgment of the

With the view we take of this case, we deem it unnecessary to address petitioners other
grievances.
WHEREFORE, the instant petition is DENIED and the assailed CA decision and resolution are
AFFIRMED.
Costs against petitioner.
SO ORDERED.

vs.
COURT OF APPEALS, respondent
G.R. No. 90580 April 8, 1991
FACTS:
A collection suit with preliminary attachment was filed by Equitable Banking
Corporation against Freeman, Inc. and Saw Chiao Lian, its President and General Manager. The
petitioners moved to intervene, alleging that (1) the loan transactions between Saw Chiao Lian
and Equitable Banking Corp. were not approved by the stockholders representing at least 2/3 of
corporate capital; (2) Saw Chiao Lian had no authority to contract such loans; and (3) there was
collusion between the officials of Freeman, Inc. and Equitable Banking Corp. in securing the
loans. The motion to intervene was denied, and the petitioners appealed to the Court of Appeals.
Meanwhile, Equitable and Saw Chiao Lian entered into a compromise agreement
which they submitted to and was approved by the lower court. But because it was not complied
with, Equitable secured a writ of execution, and two lots owned by Freeman, Inc. were levied
upon and sold at public auction to Freeman Management and Development Corp.
The Court of Appeals sustained the denial of the petitioners' motion for intervention,
holding that "the compromise agreement between Freeman, Inc., through its President, and
Equitable Banking Corp. will not necessarily prejudice petitioners whose rights to corporate
assets are at most inchoate, prior to the dissolution of Freeman, Inc. And intervention under Sec.
2, Rule 12 of the Revised Rules of Court is proper only when one's right is actual, material,
direct and immediate and not simply contingent or expectant."
It also ruled against the petitioners' argument that because they had already filed a
notice of appeal, the trial judge had lost jurisdiction over the case and could no longer issue the
writ of execution.
ISSUE:
Whether or not the Honorable Court of Appeals erred in holding that the petitioners
cannot intervene in Civil Case No. 88-44404 because their rights as stockholders of Freeman
are merely inchoate and not actual, material, direct and immediate prior to the dissolution of the
corporation.
RULING:
NO.
The petitioners base their right to intervene for the protection of their interests as
stockholders on Everett v. Asia Banking Corp. where it was held: The well-known rule that
shareholders cannot ordinarily sue in equity to redress wrongs done to the corporation, but that
the action must be brought by the Board of Directors, has its exceptions.
Equitable demurs, contending that the collection suit against Freeman, Inc, and Saw
Chiao Lian is essentially in personam and, as an action against defendants in their personal
capacities, will not prejudice the petitioners as stockholders of the corporation. The Everett case
is not applicable because it involved an action filed by the minority stockholders where the board
of directors refused to bring an action in behalf of the corporation. In the case at bar, it was
Freeman, Inc. that was being sued by the creditor bank.
On the second assignment of error, Equitable maintains that the petitioners' appeal
could only apply to the denial of their motion for intervention and not to the main case because
their personality as party litigants had not been recognized by the trial court.
(14) SAW, petitioner

After examining the issues and arguments of the parties, the Court finds that the
respondent court committed no reversible error in sustaining the denial by the trial court of the
petitioners' motion for intervention.

(15) SULO NG BAYAN vs ARANETA


GR L-31061 August 17, 1976
Facts: On April 26, 19966, Sulo ng Bayan, Inc. filed an accion de revindicacion with the Court of
First Instance of Bulacan, against Gregorio Araneta Inc. (GAI), Paradise Farms Inc., Nationa
Waterworks & Sewage Authority (NAWASA), Hacienda Caretas Inc., and the Register of Deeds
of Bulacan to recover the ownership and possession of a large tract of land in San Jose del
Monte, Bulacan containing an area of 27,982,250 sq.ms., more or less, registered under the
Torrens System in the name of GAI, et. Als predecessors-in-interest (who are members of the
corporation).
On September 2, 1966, GAI filed a motion to dismiss the amended complaint on the grounds
that (1) the complaint states no cause of action; and (2) the cause of action, if any, is barred by
prescription and laches. Paradise Farms Inc. and Hacienda Caretas Inc. filed motions to
dismiss on the same grounds. NAWASA did not file any motion to dismiss. However, it pleaded
in its answer as special and affirmative defense lack of cause of action by Sulo ng Bayan Innc.
And the barring of such action by prescription and laches.
On January 24, 1967, the trial court issued an Order of dismissal, arguing among others that the
complaint states a sufficient cause of action because the subject matter of the controversy is one
of common interest to the members of the corporation who are so numerous that the present
complaint should be treated as a class suit. The motion was denied by the trial court in its Order
dated 22 February 1967.
Sulo ng Bayan appealed to the Court of Appeals. On 3 September 1969, the Court of Appeals,
upon finding that no question of fact was involved in the appeal but only questions of law and
jurisdiction, certified the case to the Supreme Court for resolution of the legal issues involved in
the controversy.
Issues: Whether the corporation (non-stock) may institute an action in behalf of its individual
members for the recovery of certain parcels of land allegedly owned by said members, among
others.
Held: It is a doctrine well-established and obtains both at law and in equity that a corporation is
a distinct legal entity to be considered as separate and apart from the stockholders or members
who compose it, and is not affected by the personal rights, obligations and transactions of its
stockholders or members.
The property of the corporation is its property and not that of the stockholders, as owners,
although they have equities in it. Properties registered in the name of the corporation are owned
by it as an entity separate and distinct from its members. Conversely, a corporation, even in the
case of a one-man corporation.
The mere fact that one is president of a corporation does not render the property which he owns
or possesses the property of the corporation, since the president, as individual, and the
corporation are separate similatities. Similarly, stockholders in a corporation engaged in buying
and dealing in real estate whose certificates of stock entitled the holder thereof to an allotment in
the distribution of the land of the corporation upon surrender of the stock certificated were
considered not to have such legal or equitable title or interest in the land, as would support a suit
for title, especially against parties other than the corporation.

It must be noted, however, that the juridical personality of the corporation, as separate and
distinct from the persons composing it, is but a legal fiction introduced for the purpose of
convenience
This separate personality of the corporation may be disregarded, or the veil of corporate fiction
pierced, in cases where it is used as a cloak or cover for fraud or illegality, or to work an
injustice, or where necessary to achieve equity. It has not been claimed that the members have
assigned or transferred whatever rights they may have on the land in question to the
corporation. Absent any showing of interest, therefore, a corporation, has no personality to bring
the action for and in behalf of its stockholders or members for the purpose of recovering property
which belongs to said stockholders or members in their personal capacities.

(16) Adelio C. Cruz vs Quiterio L. Dalisay


Adm. Matter No. R-181-P July 31, 1987
Administrative Matter in the Supreme Court.
Doctrine: Doctrine of Piercing the Veil of Corporate Entity
A corporation has a personality distinct and separate from its individual stockholders
or members.
Facts:
A sworn complaint dated July 23, 1984 was filed by Adelio Cruz charging Quiterio Dalisay,
Senior Deputy Sheriff of Manila, with malfeasance in office, corrupt practices and serious
irregularities allegedly committed as follows:
a. Respondent attached and/or levied the money belonging to complainant Cruz when he was
not himself the judgment debtor in the final judgment of an NLRC case sought to be enforced
but rather the company known as Qualitrans Limousine Service, Inc.; and
b. Respondent also caused the service of the alias writ of execution upon complainant who is a
resident of Pasay City, despite knowledge that his territorial jurisdiction covers Manila only and
does not extend to Pasay City.
Respondent in his reply explained that when he garnished complainants cash deposit at the
Philtrust bank he was merely performing a ministerial duty. And that while it is true that said writ
was addressed to Qualitrans Limousine Service, Inc., it is also a fact that complainant had
executed an affidavit before the Pasay City assistant fiscal stating that he is the owner/ president
of Qualitrans. Because of that declaration, the counsel for the plaintiff in the labor case advised
him to serve notice of garnishment on the Philtrust bank.
Issue: Whether or not the personal property of Cruz (complainant) is properly levied or attached
as owner of the corporation?

Held: NO
Respondents actuation in enforcing a judgment against complainant who is not a judgment
debtor in the case calls for disciplinary action. What is incumbent upon respondent is to ensure
that only the portion of a decision ordained or decreed in the dispositive part should be the
subject of the execution. The tenor of the NLRC judgment and the implementing writ is clear
enough. It directed Qualitrans Limousine Service, Inc. in its judgment and not the owner thereof.
Respondent, however, choose to pierce the veil of corporate entity usurping a power
belonging to the court and assumed improvidently that since the complainant is the

owner/president of Qualitrans Limousine Service, Inc., they are one and the same. It is a well
settled doctrine both in law and equity that as a legal entity, a corporation has a personality
distinct and separate from its individual stockholders or members.
The mere fact that one is president of the corporation does not render the property he owns or
possesses the property of the corporation, since that president, as an individual, and the
corporation are separate entities.

(17) Santos vs. National Labor Relations Commission [GR 101699, 13 March 1996]
?, Vitug (J): 4 concur
Facts: Melvin D. Millena, on 1 October 1985, was hired to be the project accountant for Mana
Mining and Development Corporation's (MMDC) mining operations in Gatbo, Bacon, Sorsogon.
On 12 August 1986, Millena sent to Mr. Gil Abao, the MMDC corporate treasurer, a
memorandum calling the latter's attention to the failure of the company to comply with the
withholding tax requirements of, and to make the corresponding monthly remittances to, the
Bureau of Internal Revenue (BIR) on account of delayed payments of accrued salaries to the
company's laborers and employees.
In a letter, dated 8 September 1986, Abao advised Millena that it was the board's decision that
it stop production (operation) in Sorsogon due to the upcoming rainy seasons and the
deterioration of the peace and order in the said area; that the corporation will undertake only
necessary maintenance and repair work and will keep overhead down to the minimum
manageable level; and that the corporation will not need a project accountant until the
corporation resumes full-scale operations. Millena expressed "shock" over the termination of his
employment. He complained that he would not have resigned from the Sycip, Gores & Velayo
accounting firm, where he was already a senior staff auditor, had it not been for the assurance of
a "continuous job" by MMDC's Eng. Rodillano E. Velasquez. Millena requested that he be
reimbursed the "advances" he had made for the company and be paid his "accrued
salaries/claims." The claim was not heeded.
On October 1986, Millena filed with the NLRC Regional Arbitration, Branch No. V, in Legazpi
City, a complaint for illegal dismissal, unpaid salaries, 13 th month pay, overtime pay, separation
pay and incentive leave pay against MMDC and its two top officials, namely, Benjamin A Santos
(the President) and Rodillano A. Velasquez (the executive vice-president). In his complaintaffidavit (position paper), submitted on 27 October 1986, Millena alleged, among other things,
that his dismissal was merely an offshoot of his letter of 12 August 1986 to Abao about the
company's inability to pay its workers and to remit withholding taxes to the BIR.
On 27 July 1988, Labor Arbiter Fructouso T. Aurellano, finding no valid cause for terminating
complaint's employment, ruledthat a partial closure of an establishment due to losses was a
retrenchment measure that rendered the employer liable for unpaid salaries and other monetary
claims. The Labor Arbiter ordered Santos, et. al. to pay Millena the amount of P37,132.25
corresponding to the latter's unpaid salaries and advances: P5,400.00 for petitioner's 13th
month pay; P3,340.95 as service incentive leave pay; and P5, 400.00 as separation pay.
Santos, et. al. were further ordered to pay Millena 10% of the monetary awards as attorney's
fees. Alleging abuse of discretion by the Labor Arbiter, the company and its co-respondents filed
a "motion for reconsideration and /or appeal." 8 The motion/appeal was forthwith indorsed to the
Executive Director of the NLRC in Manila. In a resolution, dated 04 September 1989, the NLRC
affirmed the decision of the Labor Arbiter.
A writ of execution correspondingly issued; however, it was returned unsatisfied for the failure of
the sheriff to locate the offices of the corporation in the addressed indicated. Another writ of
execution and an order of garnishment was thereupon served on Santos at his residence.
Contending that he had been denied due process, Santos filed a motion for reconsideration of
the NLRC's resolution along with a prayer for the quashal of the writ of execution and order of
garnishment. He averred that he had never received any notice, summons or even a copy of the
complaint; hence, he said, the Labor Arbiter at no time had acquired jurisdiction over him. On 16
August 1991, the NLRC dismissed the motion for reconsideration. Santos filed the petition for
certiorari.
Issue: Whether Santos should be made solidarily liable with MMDC.

Held: A corporation is a judicial entity with legal personality separated and distinct from those
acting for and in its behalf and, in general, from the people comprising it. The rule is that
obligations incurred by the corporation, acting through its directors, officers and employees, are
its sole liabilities. Nevertheless, being a mere fiction of law, peculiar situations or valid grounds
can exist to warrant, albeit done sparingly, the disregard of its independent being and the lifting
of the corporate veil. As a rule, this situation might arise a corporation is used to evade a just
and due obligation or to justify a wrong, to shield or perpetrate fraud, to carry out similar other
unjustifiable aims or intentions, or as a subterfuge to commit injustice and so circumvent the law.
Without necessarily piercing the veil of corporate fiction, personal civil liability can also be said to
lawfully attach to a corporate director, trustee or officer; to wit: When (1) He assents (a) to a
patently unlawful act of the corporation, or (b) for bad faith or gross negligence in directing its
affairs, or (b) for conflict of interest, resulting in damages to the corporation, its stockholders or
other persons; (2) He consents to the issuance of watered stocks or who, having knowledge
thereof, does not forthwith file with the corporate secretary his written objection thereto; (3) He
agrees to hold himself personally and solidarily liable with the corporation; or (4) He is made, by
a specific provision of law, to personally answer for his corporate action. The case of Santos is
way of these exceptional instances. It is not even shown that Santos has had a direct hand in
the dismissal of Millena enough to attribute to Santos a patently unlawful act while acting for the
corporation. Neither can Article 289 of the Labor Code be applied since this specifically refers
only to the imposition of penalties under the Code. It is undisputed that the termination of
Millena's employment has, instead, been due, collectively, to the need for a further mitigation of
losses, the onset of the rainy season, the insurgency problem, in Sorsogon and the lack of funds
to further support the mining operation in Gatbo. It is basic that a corporation is invested by law
with a personally separate and distinct from those of the persons composing it as well as from
that of any, other legal entity to which it may be related. Mere ownership by a single stockholder
or by another corporation of all nearly all of the capital stock of a corporation is not of itself
sufficient ground for disregarding the separate corporate personally. Similar to the case of Sunio
vs. National Labor Relations Commission, Santos should not have been made personally
answerable for the payment of Millena's back salaries.

(17) BENJAMIN SANTOS


vs.
NLRC, MELVIN MILLENA
GR 101699, 13 March 1996
FACTS:
Melvin Millena, on 01 October 1985, was hired to be the project accountant for Mana
Mining and Development Corp.s mining operations in Gatbo, Bacon, Sorsogon. On 12 August
1986, private respondent sent to Mr. Gil Abao, the MMDC corporate treasurer, a memorandum
calling the latter's attention to the failure of the company to comply with the withholding tax
requirements of, and to make the corresponding monthly remittances to, the Bureau of Internal
Revenue ("BIR") on account of delayed payments of accrued salaries to the company's laborers
and employees.
Albao responded that the mining operations in Sorsogon shall be stopped pending
the end of the wet season and the normalization of the peace and order situation in the province.
Therefore, MMDC is dispensing the services of Millena because of lack of work load.
Private respondent expressed "shock" over the termination of his employment. He
complained that he would not have resigned from the Sycip, Gorres & Velayo accounting firm,
where he was already a senior staff auditor, had it not been for the assurance of a "continuous
job" by MMDC's Engr. Rodillano E. Velasquez. Private respondent requested that he be
reimbursed the "advances" he had made for the company and be paid his "accrued
salaries/claims
With his demands left unheeded, Millena filed a complaint for illegal dismissal, unpaid
salaries, 13th month pay, overtime pay, separation pay and incentive leave pay against MMDC
and its two top officials, namely, herein petitioner Benjamin A. Santos (the President) and
Rodillano A. Velasquez (the executive vice-president).
ISSUE:
Whether or not the impleaded officials of MMDC may be held liable.
RULING:
NO.
It was not proven that they acted in bad faith. A corporation is a juridical entity with
legal personality separate and distinct from those acting for and in its behalf and, in general,
from the people comprising it. Nevertheless, being a mere fiction of law, peculiar situations or
valid grounds can exist to warrant, albeit done sparingly, the disregard of its independent being
and the lifting of the corporate veil. The Court also has collated the settled instances when,
without necessarily piercing the veil of corporate fiction, personal civil liability can also be said to
lawfully attach to a corporate director, trustee or officer.
The case of petitioner is way off these exceptional instances. It is not even shown that
petitioner has had a direct hand in the dismissal of private respondent enough to attribute to him
(petitioner) a patently unlawful act while acting for the corporation. It is undisputed that the
termination of petitioner's employment has, instead, been due, collectively, to the need for a
further mitigation of losses, the onset of the rainy season, the insurgency problem in Sorsogon
and the lack of funds to further support the mining operation in Gatbo.

(18) G.R. No. L-15121

August 31, 1962

GREGORIO PALACIO, in his own behalf and in behalf of his minor child,
MARIO PALACIO, plaintiffs-appellants,
vs.
FELY TRANSPORTATION COMPANY, defendant-appellee.
This is an appeal by the plaintiffs from the decision of the Court of First Instance of Manila which
dismissed their complaint.
Originally taken to the Court of Appeals, this appeal was certified to this Court on the ground that
it raises purely questions of law.
The parties in this case adopt the following findings of fact of the lower court:
In their complaint filed with this Court on May 15, 1954, plaintiffs allege, among other
things, "that about December, 1952, the defendant company hired Alfredo Carillo as
driver of AC-787 (687) (a registration for 1952) owned and operated by the said
defendant company; that on December 24, 1952, at about 11:30 a.m., while the driver
Alfonso (Alfredo) Carillo was driving AC-687 at Halcon Street, Quezon City, wilfully,
unlawfully and feloniously and in a negligent, reckless and imprudent manner, run
over a child Mario Palacio of the herein plaintiff Gregorio Palacio; that on account of
the aforesaid injuries, Mario Palacio suffered a simple fracture of the right tenor (sic),
complete third, thereby hospitalizing him at the Philippine Orthopedic Hospital from
December 24, 1952, up to January 8, 1953, and continued to be treated for a period of
five months thereafter; that the plaintiff Gregorio Palacio herein is a welder by
occupation and owner of a small welding shop and because of the injuries of his child
he has abandoned his shop where he derives income of P10.00 a day for the support
of his big family; that during the period that the plaintiff's (Gregorio Palacio's) child was
in the hospital and who said child was under treatment for five months in order to meet
the needs of his big family, he was forced to sell one air compressor (heavy duty) and
one heavy duty electric drill, for a sacrifice sale of P150.00 which could easily sell at
P350.00; that as a consequence of the negligent and reckless act of the driver Alfredo
Carillo of the herein defendant company, the herein plaintiffs were forced to litigate this
case in Court for an agreed amount of P300.00 for attorney's fee; that the herein
plaintiffs have now incurred the amount of P500.00 actual expenses for transportation,
representation and similar expenses for gathering evidence and witnesses; and that
because of the nature of the injuries of plaintiff Mario Palacio and the fear that the
child might become a useless invalid, the herein plaintiff Gregorio Palacio has suffered
moral damages which could be conservatively estimated at P1,200.00.
On May 23, 1956, defendant Fely Transportation Co., filed a Motion to Dismiss on the
grounds (1) that there is no cause of action against the defendant company, and (2)
that the cause of action is barred by prior judgment..
In its Order, dated June 8, 1956, this Court deferred the determination of the grounds
alleged in the Motion to Dismiss until the trial of this case.
On June 20, 1956, defendant filed its answer. By way of affirmative defenses, it

alleges (1) that complaint states no cause of action against defendant, and (2) that the
sale and transfer of the jeep AC-687 by Isabelo Calingasan to the Fely Transportation
was made on December 24, 1955, long after the driver Alfredo Carillo of said jeep had
been convicted and had served his sentence in Criminal Case No. Q-1084 of the
Court of First Instance of Quezon City, in which both the civil and criminal cases were
simultaneously tried by agreement of the parties in said case. In the Counterclaim of
the Answer, defendant alleges that in view of the filing of this complaint which is a
clearly unfounded civil action merely to harass the defendant, it was compelled to
engage the services of a lawyer for an agreed amount of P500.00.
During the trial, plaintiffs presented the transcript of the stenographic notes of the trial
of the case of "People of the Philippines vs. Alfredo Carillo, Criminal Case No. Q1084," in the Court of First Instance of Rizal, Quezon City (Branch IV), as Exhibit
"A".1wph1.t
It appears from Exhibit "A" that Gregorio Palacio, one of the herein plaintiffs, testified
that Mario Palacio, the other plaintiff, is his son; that as a result of the reckless driving
of accused Alfredo Carillo, his child Mario was injured and hospitalized from
December 24, 1952, to January 8, 1953; that during all the time that his child was in
the hospital, he watched him during the night and his wife during the day; that during
that period of time he could not work as he slept during the day; that before his child
was injured, he used to earn P10.00 a day on ordinary days and on Sundays from
P20 to P50 a Sunday; that to meet his expenses he had to sell his compressor and
electric drill for P150 only; and that they could have been sold for P300 at the lowest
price.
During the trial of the criminal case against the driver of the jeep in the Court of First
Instance of Quezon City (Criminal Case No. Q-1084) an attempt was unsuccessfully
made by the prosecution to prove moral damages allegedly suffered by herein plaintiff
Gregorio Palacio. Likewise an attempt was made in vain by the private prosecutor in
that case to prove the agreed attorney's fees between him and plaintiff Gregorio
Palacio and the expenses allegedly incurred by the herein plaintiffs in connection with
that case. During the trial of this case, plaintiff Gregorio Palacio testified substantially
to the same facts.
The Court of First Instance of Quezon City in its decision in Criminal Case No. 1084
(Exhibit "2") determined and thoroughly discussed the civil liability of the accused in
that case. The dispositive part thereof reads as follows:
IN VIEW OF THE FOREGOING, the Court finds the accused Alfredo Carillo y Damaso
guilty beyond reasonable doubt of the crime charged in the information and he is
hereby sentenced to suffer imprisonment for a period of Two Months & One Day of
Arresto Mayor; to indemnify the offended party, by way of consequential damages, in
the sum of P500.00 which the Court deems reasonable; with subsidiary imprisonment
in case of insolvency but not to exceed /3 of the principal penalty imposed; and to pay
the costs.
On the basis of these facts, the lower court held action is barred by the judgment in the criminal
case and, that under Article 103 of the Revised Penal Code, the person subsidiarily liable to pay
damages is Isabel Calingasan, the employer, and not the defendant corporation.
Against that decision the plaintiffs appealed, contending that:

THE LOWER COURT ERRED IN NOT SUSTAINING THAT THE DEFENDANTAPPELLEE IS SUBSIDIARILY LIABLE FOR DAMAGES AS A RESULT OF CRIMINAL
CASE NO. Q-1084 OF THE COURT OF FIRST INSTANCE OF QUEZON CITY FOR
THE REASON THAT THE INCORPORATORS OF THE FELY TRANSPORTATION
COMPANY, THE DEFENDANT-APPELLEE HEREIN, ARE ISABELO CALINGASAN
HIMSELF, HIS SON AND DAUGHTERS;
THE LOWER COURT ERRED IN NOT CONSIDERING THAT THE INTENTION OF
ISABELO CALINGASAN IN INCORPORATING THE FELY TRANSPORTATION
COMPANY, THE DEFENDANT-APPELLEE HEREIN, WAS TO EVADE HIS CIVIL
LIABILITY AS A RESULT OF THE CONVICTION OF HIS DRIVER OF VEHICLE AC687 THEN OWNED BY HIM:
THE LOWER COURT ERRED IN HOLDING THAT THE CAUSE OF ACTION OF THE
PLAINTIFFS-APPELLANTS IS BARRED BY PRIOR JUDGMENT.
With respect to the first and second assignments of errors, plaintiffs contend that the defendant
corporate should be made subsidiarily liable for damages in the criminal case because the sale
to it of the jeep in question, after the conviction of Alfred Carillo in Criminal Case No. Q-1084 of
the Court of First Instance of Quezon City was merely an attempt on the part of Isabelo
Calingasan its president and general manager, to evade his subsidiary civil liability.
The Court agrees with this contention of the plaintiffs. Isabelo Calingasan and defendant Fely
Transportation may be regarded as one and the same person. It is evident that Isabelo
Calingasan's main purpose in forming the corporation was to evade his subsidiary civil
liability1 resulting from the conviction of his driver, Alfredo Carillo. This conclusion is borne out by
the fact that the incorporators of the Fely Transportation are Isabelo Calingasan, his wife, his
son, Dr. Calingasan, and his two daughters. We believe that this is one case where the
defendant corporation should not be heard to say that it has a personality separate and distinct
from its members when to allow it to do so would be to sanction the use of the fiction of
corporate entity as a shield to further an end subversive of justice. (La Campana Coffee Factory,
et al. v. Kaisahan ng mga Manggagawa, etc., et al., G.R. No. L-5677, May 25, 1953)
Furthermore, the failure of the defendant corporation to prove that it has other property than the
jeep (AC-687) strengthens the conviction that its formation was for the purpose above indicated.
And while it is true that Isabelo Calingasan is not a party in this case, yet, is held in the case of
Alonso v. Villamor, 16 Phil. 315, this Court can substitute him in place of the defendant
corporation as to the real party in interest. This is so in order to avoid multiplicity of suits and
thereby save the parties unnecessary expenses and delay. (Sec. 2, Rule 17, Rules of Court;
Cuyugan v. Dizon. 79 Phil. 80; Quison v. Salud, 12 Phil. 109.)
Accordingly, defendants Fely Transportation and Isabelo Calingasan should be held subsidiarily
liable for P500.00 which Alfredo Carillo was ordered to pay in the criminal case and which
amount he could not pay on account of insolvency.
We also sustain plaintiffs' third assignment of error and hold that the present action is not barred
by the judgment of the Court of First Instance of Quezon City in the criminal case. While there
seems to be some confusion on part of the plaintiffs as to the theory on which the is based
whether ex-delito or quasi ex-delito (culpa aquiliana) We are convinced, from the discussion
prayer in the brief on appeal, that they are insisting the subsidiary civil liability of the defendant.
As a matter of fact, the record shows that plaintiffs merely presented the transcript of the
stenographic notes (Exhibit "A") taken at the hearing of the criminal case, which Gregorio
Palacio corroborated, in support of their claim for damages. This rules out the defense of res
judicata, because such liability proceeds precisely from the judgment in the criminal action,

where the accused was found guilty and ordered to pay an indemnity in the sum P500.00.
WHEREFORE, the decision of the lower court is hereby reversed and defendants Fely
Transportation and Isabelo Calingasan are ordered to pay, jointly and severally, the plaintiffs the
amount of P500.00 and the costs.
(18) G.R. No. L-15121, August 31, 1962
Gregorio Palacio and Mario Palacio (minor)
vs Fely Transportation Company
Ponente: Regala
Facts:
In their complaint, the Palacio alleged that Fely hired Alfredo Canillo as driver who negligently
run over a child (Mario). Gregorio , the father of Mario is a welder and in the account of his
child's injuries has abandoned his shop which is the family's source of income.
Fely filed a motion to dismiss on the grounds that there is no cause of action against the
company and that the cause of action is barred by prior judgment. But the court deferred the
determination of the grounds alleged in the motion to dismiss until the trial of the case.
The defendant then alleges (1) that complaint states no cause of action against defendant, and
(2) that the sale and transfer of the jeep AC-687 by Isabelo Calingasan to the Fely
Transportation was made on December 24, 1955, long after the driver Alfredo Carillo of said
jeep had been convicted and had served his sentence.
In view of the evidence presented, the lower court barred the judgment in the criminal case and
held that the person subsidiarily liable to pay damages is Isabel Calingasan, the employer.
Issue: Whether Fely Transportation can be held liable for the damages.
Ruling:
The Court agrees with this contention of the plaintiffs. Isabelo Calingasan and defendant Fely
Transportation may be regarded as one and the same person. It is evident that Isabelo
Calingasan's main purpose in forming the corporation was to evade his subsidiary civil liability
resulting from the conviction of his driver, Alfredo Carillo. This conclusion is borne out by the fact
that the incorporators of the Fely Transportation are Isabelo Calingasan, his wife, his son, Dr.
Calingasan, and his two daughters.
Accordingly, defendants Fely Transportation and Isabelo Calingasan should be held subsidiarily
liable for P500.00 which Alfredo Carillo was ordered to pay in the criminal case and which
amount he could not pay on account of insolvency.

(19) G.R. No. 98185 December 11, 1992


SIBAGAT TIMBER CORPORATION, petitioner,
vs.
ADOLFO B. GARCIA, USIPHIL, INC. and STRONGHOLD INSURANCE CO.,
INC., respondents.
GRIO-AQUINO, J.:
This is a petition for review on certiorari of the decision of the Court of Appeals dated February
15, 1991 in CA-G.R. No. 20799 entitled, "Sibagat Timber Corp. vs. Adolfo B. Garcia, et al.,"
affirming the decision of the Regional Trail Court which dismissed the petitioner's petition
for certiorari, prohibition and injunction with restraining order and writ of preliminary injunction
and damages (Spl. Case No. 548, RTC, Branch I, Butuan City).
On August 30, 1988, respondent Sheriff Adolfo B. Garcia, who was entrusted with the
implementation of the writ of execution issued by the Regional Trial Court, Branch 147, Makati,
Metro Manila in Civil Case No. 7180 entitled, "USIPHIL, INC. vs. Del Rosario and Sons Logging
Enterprises, Inc.," levied on the following personal properties of Del Rosario & Sons, Inc.:
One (1) Unit CAT Grader with SN 99E-5016.
One (1) Unit Generating Set with Cummins
Engine No. 1074304 Model V-855QC and
Generator 125 KVA No. HA-90071 1720-1
and Panel Switch Board.
One (1) Generating Set with CAT D-311
Series H No. 51B4241 w/ Generator No.
30TH 211 1800 RPM, 60 Cycles, 30KVA
One (1) pc. Engine Block CAT D-4600.
One (1) TD-25B w/ Hyster D988, Triple Drum
Model BY B14 SN-9PI55E
One (1) TD-25A w/ No. Engine Number, w/
Radiator and X-2 Triple Drum Model 142
Yarder and blade.
which he scheduled for sale at public auction on September 7, 1988 at 10:00 o'clock in the
morning. He also levied on:
One (1) Unit Reo Logging Truck (5) tonner

not in running condition; and


One (1) Unit White Logging (5) tonner not in
running condition.
which he scheduled for sale at public auction on September 8, 1988.
On the same date (August 30, 1988) that levy was made by the sheriff, the petitioner herein,
through Mariano Rana, filed a third-party claim alleging that it is the lawful owner of the levied
machinery and equipment, by virtue of deeds of sale executed in its favor by Del Rosario & Sons
Logging Enterprises, Inc.
Pursuant to Section 17, Rule 39 of the Rules of Court, an indemnity bond was posted by the
judgment creditor, USIPHIL, Inc., to indemnify the respondent sheriff against the claim of the
third-party claimant.
On September 6, 1988, at 2:00 P.M., petitioner filed in the Regional Trial Court of Butuan City, a
petition for "Certiorari, Prohibition and Injunction with Restraining Order & Writ of Preliminary
Injunction and Damages" in Special Civil Case No. 548. A temporary restraining order was
issued on September 6, 1988 by the Executive Judge of that court.
On September 7, 1988, at 11:10 A.M., the court employees who were deputized to serve the
restraining order arrived at the place where the auction sale was to be held. However, they were
told by sheriff Garcia that the auction sale was finished at 10:30 A.M. yet, and that a certificate of
sale for each of the personal properties to be auctioned on that day had already been issued to
USIPHIL, INC., the judgment creditor, as the only bidder and purchaser.
After the hearing on the application for preliminary injunction was held on September 15, 1988,
the parties were directed to submit simultaneous memoranda. Thereafter the case was deemed
submitted for resolution. In the meantime, respondent USIPHIL, INC., filed a formal motion to
dismiss the petition which the trial court granted on February 28, 1990.
On March 9, 1990, the petitioner appealed the order of dismissal to the Court of Appeals (CA
G.R. No. 20799). On February 15, 1991, the Court of Appeals dismissed the appeal.
Petitioner's motion for reconsideration was denied by the Court of Appeals. Hence, this petition
for review under Rule 45 of the Rules of Court.
The main issue raised by the petitioner is the supposed error of the Court of Appeals in piercing
the veil of corporate entity and in holding that the third-party claimant, herein petitioner Sibagat
Corporation, is not a separate and distinct entity from the judgment debtor, Del Rosario & Sons
Logging Enterprises, Inc.
As pointed out by the Court of Appeals in its decision:
Gleaned from the records of this case, Mariano Rana, the third-party
claimant for and in behalf of petitioner testified, among others, that he is the
office manager of Sibagat Timber Corporation (p. 58, Record); that he is the
administrative manager of Del Rosario and Sons Logging Enterprises, Inc.
in a concurrent capacity (p. 50, id. ); that the officers of the Sibagat Timber
Corporation are: Mr. Policarpio C. Del Rosario, President and General

Manager; Miss Conchita C. Del Rosario, Vice-President and General


Manager (p. 60, Id.); and the Directors are: Policarpio Del Rosario, Jr.,
Cristina Del Rosario, Mrs. Jasmin Del Rosario, and Vicente C. Cel Rosario
(pp. 61-63, id.). On the part of Del Rosario and Sons Logging Enterprises,
Inc., the officers of the company are: Mr. Policarpio C. Cel Rosario,
President;
Miss
Conchita
Del
Rosario,
Vice-President/General
Manager/Director and Treasurer; Mrs. Jasmin A. Del Rosario, Querubin Del
Rosario, and Cristeta Del Rosario, respectively. (p. 29, Rollo.)
The circumstances that: (1) petitioner and Del Rosario & Sons Logging Enterprises, Inc. hold
office in the same building; (2) the officers and directors of both corporations are practically the
same; and (3) the Del Rosarios assumed management and control of Sibagat and have been
acting for and managing its business (p. 30, Rollo), bolster the conclusion that petitioner is an
alter ego of the Del Rosario & Sons Logging Enterprises, Inc.
The rule is that the veil of corporate fiction may be pierced when made as a shield to perpetrate
fraud and/or confuse legitimate issues (Jacinto vs. CA, 198 SCRA 211). The theory of corporate
entity was not meant to promote unfair objectives or otherwise, to shield them (Villanueva vs.
Adre, 172 SCRA 876). Likewise, where it appears that two business enterprises are owned,
conducted, and controlled by the same parties, both law and equity will, when necessary to
protect the rights of third persons, disregard the legal fiction that two corporations are distinct
entities, and treat them as identical (Phil. Veterans Investment Development Corp. vs. CA, 181
SCRA 669).
The petitioner further contends that the Court of Appeals erroneously disregarded the decision of
this Court in G.R. No. 84497 entitled, "Alfonso Escovilla, Jr., Cecilio M. Meris and Cuison
Engineering and Machinery Co., Inc., Petitioner vs. The Hon. Court of Appeals, Sibagat Timber
Corporation and Conchita del Rosario, Respondents," wherein this Court held that private
respondents (herein petitioner) are the actual owners of the properties subject of execution by
virtue of a sale in their favor by Del Rosario & Sons Logging Enterprises, Inc.
That allegation has no merit. The issue raised in that case was "whether or not an action for
prohibition will prosper as a remedy for acts already accomplished." It was a procedural
question, not the ownership of the properties subject of the execution.
The issue of ownership being raised now by the petitioner involves a factual question requiring
an assessment of the evidence. This may not be in a petition for review under Rule 45 for it is
not the function of this Court to examine and weigh evidence already considered in the
proceedings below. Our jurisdiction is limited to reviewing only errors of law that may have been
committed by the lower courts (Navarra vs. CA, 204 SCRA 850).
Assuming arguendo that this Court in G.R. No. 84497 held that petitioner is the owner of the
properties levied under execution, that circumstance will not be a legal obstacle to the piercing
of the corporate fiction. As found by both the trial and appellate courts, petitioner is just a
conduit, if not an adjunct of Del Rosario & Sons Logging Enterprises, Inc. In such a case, the
real ownership becomes unimportant and may be disregard for the two entities may/can be
treated as only one agency or instrumentality.
The corporate entity is disregarded where a corporation is the mere alter
ego, or business conduit of a person or where the corporation is so
organized and controlled and its affairs are so conducted, as to make it
merely an instrumentality, agency, conduit or adjunct of another corporation.
(Aguedo F. Agbayani Commercial Laws of the Philippines, Vol. 3, 1984 Ed.,
p. 30, citing decided cases.)

WHEREFORE, the petition for review is DENIED and the decision of the Court of Appeals is
AFFIRMED.

fraud, or defend crime or where a corporation is the mere alter ego or


business conduit of a person. To disregard the separate juridical personality
of a corporation, the wrongdoing must be clearly and convincingly
established. It cannot be presumed.
(21) CONCEPT BUILDERS, INC.
vs.
THE NATIONAL LABOR RELATIONS COMMISSION
G.R. No. 108734. May 29, 1996

SO ORDERED.

(20) YU VS. NLRC, 16 JUNE 1995


NATURE:
FACTS:

Petition for certiorari to review a decision of the NLRC


Private respondents were employees of respondent corporation Tanduay
Distillery, Inc, (TDI). They were among the 22 employees of TDI who received a
memorandum from TDI terminating their services for reasons of retrenchment. A
buyer of TDI's assets, Twin Ace Holdings, Inc. took over the business. Twin Ace
assumed the business name Tanduay Distillers. Employees filed a motion to
implead petitioners James Yu and Wilson Young, doing business under the
name and style of Tanduay Distillers as party respondents.
After declaring that the retrenchment was illegal, public respondent NLRC
issued two orders of execution. In its first order dated 24 May 1989, only TDI
was held liable to reinstate respondents up to the time of change of ownership,
and for separation benefits. In the second one dated 17 November 1992
petitioners and Tanduay Distillers were ordered to reinstate private respondents
employees.
Petitioners prayer: That the motion for execution ordering petitioners to
reinstate private respondents be denied because Tanduay Distillers is an entity
distinct and separate from TDI, the previous owner
Private respondents prayer: That they be reinstated by Tanduay Distillers
NLRCs ruling: That petitioners and Tanduay Distillers reinstate private
respondents

ISSUE:

WON to pierce the veil of corporate entity of Tanduay Distillers and hold its
president solidarily liable for quasi-delict

RULING:

Petition granted, NLRCs ruling reversed.


TWIN ACE OR TANDUAY DISTILLERS, ON ONE HAND, AND TANDUAY DISTILLERY
INC. (TDI), ON THE OTHER, ARE DISTINCT AND SEPARATE CORPORATIONS. The
name of the company for whom the petitioners are working is Twin Ace
Holdings Corporation. Moreover, Twin Ace is part of the Allied Bank Group
although it conducts the rum business under the name of Tanduay Distillers.
The use of a similar sounding or almost identical name is an obvious device
to capitalize on the goodwill, which Tanduay Rum has built over the years.
There is nothing to suggest that the owners of TDI, have any common
relationship as to identify it with Allied Bank Group which runs Tanduay
Distillers. The genuine nature of the sale to Twin Ace is evidenced by the
fact that Twin Ace was only a subsequent interested buyer. At the time when
termination notices were sent to its employees, TDI was negotiating with the
First Pacific Metro Corporation for the sale of its assets. Only after First
Pacific gave up its efforts to acquire the assets did Twin Ace or Tanduay
Distillers come into the picture. No proof was presented as regards the
communality of ownership and management to support the contention that
the two companies are one firm or closely related.
The doctrine of piercing the veil of corporate entity applies when the
corporate fiction is used to defeat public convenience, justify wrong, protect

FACTS:
Petitioner, a domestic corporation, with principal office at 355 Maysan Road,
Valenzuela, Metro Manila, is engaged in the construction business. Private respondents were
employed by said company as laborers, carpenters and riggers.
Eventually, respondents services were terminated. The Labor Arbiter then rendered
judgment ordering petitioner to reinstate private respondents and to pay them back wages. A writ
of execution was then issued but was partially satisfied because the sheriff reported all the
employees inside petitioner's premises at 355 Maysan Road, Valenzuela, Metro Manila, claimed
that they were employees of Hydro Pipes Philippines, Inc and not by respondent. Subsequently,
a certain Dennis Cuyegkeng filed a third-party claim with the Labor Arbiter alleging that the
properties sought to be levied upon by the sheriff were owned by Hydro (Phils.), Inc. of which he
is the Vice-President.
Private respondents filed a "Motion for Issuance of a Break-Open Order," alleging that
HPPI and petitioner corporation were owned by the same incorporator/stockholders. They also
alleged that petitioner temporarily suspended its business operations in order to evade its legal
obligations to them and that private respondents were willing to post an indemnity bond to
answer for any damages which petitioner and HPPI may suffer because of the issuance of the
break-open order.
ISSUE:
Whether or not petitioner corporation and HPPI are one and the same.
RULING:
YES.
It is a fundamental principle of corporation law that a corporation is an entity separate
and distinct from its stockholders and from other corporations to which it may be connected.
But, this separate and distinct personality of a corporation is merely a fiction created by law for
convenience and to promote justice. So, when the notion of separate juridical personality is
used to defeat public convenience, justify wrong, protect fraud or defend crime, or is used as a
device to defeat the labor laws, this separate personality of the corporation may be disregarded
or the veil of corporate fiction pierced. This is true likewise when the corporation is merely an
adjunct, a business conduit or an alter ego of another corporation.
The test in determining the applicability of the doctrine of piercing the veil of corporate
fiction is as follows: a.
Control, not mere majority or complete stock control, but complete
domination, not only of finances but of policy and business practice in respect to the transaction
attacked so that the corporate entity as to this transaction had at the time no separate mind, will
or existence of its own; b.
Such control must have been used by the defendant to commit
fraud or wrong, to perpetuate the violation of a statutory or other positive legal duty or dishonest
and unjust act in contravention of plaintiff's legal rights; and c.
The aforesaid control
and breach of duty must proximately cause the injury or unjust loss complained of.
HPPI is obviously a business conduit of Petitioner Corporation and its emergence was
skillfully orchestrated to avoid the financial liability that already attached to Petitioner
Corporation.

(21) Concept Builders Inc. vs. National Labor Relations Commission (NLRC, First
Division) [GR
108734, 29 May 1996]
First Division, Hermosisima Jr. (J): 4 concur
Facts: Concept Builders, Inc., (CBI) a domestic corporation, with principal office at 355 Maysan
Road, Valenzuela, Metro Manila, is engaged in the construction business while Norberto
Marabe; Rodolfo Raquel, Cristobal Riego, Manuel Gillego, Palcronio Giducos, Pedro Aboigar,
Norberto Comendador, Rogelio Salut, Emilio Garcia, Jr., Mariano Rio, Paulina Basea, Alfredo
Albera, Paquito Salut, Domingo Guarino, Romeo Galve, Dominador Sabina, Felipe Radiana,
Gavino Sualibio, Moreno Escares, Ferdinand Torres, Felipe Basilan, and Ruben Robalos were
employed by said company as laborers, carpenters and riggers. On November 1981, Marabe,
et. al. were served individual written notices of termination of employment by CBI, effective on
30 November 1981. It was stated in the individual notices that their contracts of employment had
expired and the project in which they were hired had been completed. The National Labor
Relations Commission (NLRC) found it to be, the fact, however, that at the time of the
termination of Marabe, et.al.'s employment, the project in which they were hired had not yet
been finished and completed. CBI had to engage the services of sub-contractors whose workers
performed the functions of Marabe, et. al. Aggrieved, Marabe, et. al. filed a complaint for illegal
dismissal, unfair labor practice and non-payment of their legal holiday pay, overtime pay and
thirteenth-month pay against CBI. On 19 December 1984, the Labor Arbiter rendered judgment
ordering CBI to reinstate Marabe et. al. and to pay them back wages equivalent to 1 year or 300
working days. On 27 November 1985, the NLRC dismissed the motion for reconsideration filed
by CBI on the ground that the said decision had already become final and executory.
On 16 October 1986, the NLRC Research and Information Department made the finding that
Marabe, et. al.'s back wages amounted to P199,800.00. On 29 October 1986, the Labor Arbiter
issued a writ of execution directing the sheriff to execute the Decision, dated 19 December 1984.
The writ was partially satisfied through garnishment of sums from CBI's debtor, the Metropolitan
Waterworks and Sewerage Authority, in the amount of P81,385.34. Said amount was turned
over to the cashier of the NLRC. On 1 February 1989, an Alias Writ of Execution was issued by
the Labor Arbiter directing the sheriff to collect from CBI the sum of P117,414.76, representing
the balance of the judgment award, and to reinstate Marabe, et. al. to their former positions. On
13 July 1989, the sheriff issued a report stating that he tried to serve the alias writ of execution
on petitioner through the security guard on duty but the service was refused on the ground that
CBI no longer occupied the premises. On 26 September 1986, upon motion of Marabe, et. al.,
the Labor Arbiter issued a second alias writ of execution. The said writ had not been enforced by
the special sheriff because, as stated in his progress report dated 2 November 1989, that all the
employees inside CBI's premises claimed that they were employees of Hydro Pipes Philippines,
Inc. (HPPI) and not by CBI; that levy was made upon personal properties he found in the
premises; and that security guards with high-powered guns prevented him from removing the
properties he had levied upon. The said special sheriff recommended that a "break-open order"
be issued to enable him to enter CBI's premises so that he could proceed with the public auction
sale of the aforesaid personal properties on 7 November 1989. On 6 November 1989, a certain
Dennis Cuyegkeng filed a third-party claim with the Labor Arbiter alleging that the properties
sought to be levied upon by the sheriff were owned by HPPI, of which he is the Vice-President.
On 23 November 1989, Marabe, et. al. filed a "Motion for Issuance of a Break-Open Order,"
alleging that HPPI and CBI were owned by the same incorporator/stockholders. They also
alleged that petitioner temporarily suspended its business operations in order to evade its legal
obligations to them and that Marabe, et. al. were willing to post an indemnity bond to answer for
any damages which CBI and HPPI may suffer because of the issuance of the break-open order.
On 2 March 1990, the Labor Arbiter issued an Order which denied Marabe, et. al.'s motion for

break-open order. Marabe, et. al. then appealed to the NLRC. On 23 April 1992, the NLRC set
aside the order of the Labor Arbiter, issued a break-open order and directed Marabe, et. al. to
file a bond. Thereafter, it directed the sheriff to proceed with the auction sale of the properties
already levied upon. It dismissed the third-party claim for lack of merit. CBI moved for
reconsideration but the motion was denied by the NLRC in a Resolution, dated 3 December
1992. Hence, the petition.
Issue: Whether the NLRC was correct in issuing the break-open order to levy the HPPI
properties located at CBI and/or HPPIs premises at 355 Maysan Road, Valenzuela, Metro
Manila.
Held: It is a fundamental principle of corporation law that a corporation is an entity separate and
distinct from its stockholders and from other corporations to which it may be connected. But, this
separate and distinct personality of a corporation is merely a fiction created by law for
convenience and to promote justice. So, when the notion of separate juridical personality is used
to defeat public convenience, justify wrong, protect fraud or defend crime, or is used as a device
to defeat the labor laws, this separate personality of the corporation may be disregarded or the
veil of corporate fiction pierced. This is true likewise when the corporation is merely an adjunct, a
business conduit or an alter ego of another corporation. The conditions under which the juridical
entity may be disregarded vary according to the peculiar facts and circumstances of each case.
No hard and fast rule can be accurately laid down, but certainly, there are some probative
factors of identity that will justify the application of the doctrine of piercing the corporate veil, to
wit: (1) Stock ownership by one or common ownership of both corporations; (2) Identity of
directors and officers; (3) The manner of keeping corporate books and records; and (4) Methods
of conducting the business.
The SEC en banc explained the "instrumentality rule" which the courts have applied in
disregarding the separate juridical personality of corporations as "Where one corporation is so
organized and controlled and its affairs are conducted so that it is, in fact, a mere instrumentality
or adjunct of the other, the fiction of the corporate entity of the "instrumentality" may be
disregarded. The control necessary to invoke the rule is not majority or even complete stock
control but such domination of instances, policies and practices that the controlled corporation
has, so to speak, no separate mind, will or existence of its own, and is but a conduit for its
principal. It must be kept in mind that the control must be shown to have been exercised at the
time the acts complained of took place. Moreover, the control and breach of duty must
proximately cause the injury or unjust loss for which the complaint is made."
The test in determining the applicability of the doctrine of piercing the veil of corporate fiction is
as (1) Control, not mere majority or complete stock control, but complete domination, not only of
finances but of policy and business practice in respect to the transaction attacked so that the
corporate entity as to this transaction had at the time no separate mind, will or existence of its
own; (2) Such control must have been used by the defendant to commit fraud or wrong, to
perpetuate the violation of a statutory or other positive legal duty or dishonest and unjust act in
contravention of plaintiff's legal rights; and (3) The aforesaid control and breach of duty must
proximately cause the injury or unjust loss complained of. The absence of any one of these
elements prevents "piercing the corporate veil." In applying the "instrumentality" or "alter ego"
doctrine, the courts are concerned with reality and not form, with how the corporation operated
and the individual defendant's relationship to that operation. Thus the question of whether a
corporation is a mere alter ego, a mere sheet or paper corporation, a sham or a subterfuge is
purely one of fact. Here, while CBI claimed that it ceased its business operations on 29 April
1986, it filed an Information Sheet with the Securities and Exchange Commission on 15 May
1987, stating that its office address is at 355 Maysan Road, Valenzuela, Metro Manila.
On the other hand, HPPI, the third-party claimant, submitted on the same day, a similar
information sheet stating that its office address is at 355 Maysan Road, Valenzuela, Metro
Manila.

Further, both information sheets were filed by the same Virgilio O. Casio as the corporate
secretary of both corporations. Both corporations had the same president, the same board of
directors, the same corporate officers, and substantially the same subscribers. From the
foregoing, it appears that, among other things, the CBI and the HPPI shared the same address
and/or premises. Under these circumstances, it cannot be said that the property levied upon by
the sheriff were not of CBI's. Clearly, CBI ceased its business operations in order to evade the
payment to Marabe, et. al. of back wages and to bar their reinstatement to their former positions.
HPPI is obviously a business conduit of CBI and its emergence was skillfully orchestrated to
avoid the financial liability that already attached to CBI.

(22) Koppel Philippines, Inc. vs Alfredo Yatco (Collector of Internal Revenue)


GR L47673, 77 Phil 496, October 10, 1946
Facts:
Koppel Industrial Car and Equipment company (KICE), a foreign company not doing business in
the Philippines, owned 995 shares out of the 1000 shares that comprise the capital stock of KPI,
a domestic corporation licensed as commercial broker in the Philippines. The remaining 5
shares were owned by each of the officers of KPI. KICE is in the business of selling railway
materials, machineries and supplies. Buyers in the Philippines, when interested, asked for price
quotations from KPI, and KPI then cabled for the quotation desired from KICE. However, KPI
quoted to the purchaser a selling price above the figures quoted by KICE. On the basis of these
quotations, orders were placed by the local buyers. Between KICE and KPI, the arrangement
nonetheless was that KICE controls how much share of the profits goes to KPI. For these
transactions, the BIR treated KPI as a subsidiary of KICE and collected from KPI the merchants
sales tax, which was a revenue law in force at the time the sales took place.
KPI paid the taxes under protest, demanded for refund and contended that KPI could not be
liable for merchants sales tax because it was only acting as broker between KICE and the local
buyers. The lower court dismissed the complaint and ruled in favor of the government.

Issue 1: W/N KPI did business with the local buyers as an agent of KICE and not as broker
Held:
Yes. The facts that KICE unilaterally controls the amount of so-called share in the profits of
KPI and that KICE owns an overwhelming majority (99.5%) of the capital stock of the KPI are
sufficient to conclude that the latter is a mere dummy, agent or wholly-owned subsidiary of KICE.
Such conclusion is based on the doctrine that courts may pierce the corporate veil to uncover
the true intents of these corporations.

Issue 2: W/N the application of piercing the corporate veil doctrine is proper
Held:
Yes. With regards only to the transactions involved, KPI and KICE were treated as one and the
same so that taxes could be rightly collected. The court has to disregard this corporate fiction
to prevent KICE / KPI from evading its taxes by contravening the local internal revenue laws.
The court did not deny legal personality to KPI; in fact, it had no power to hold so. The doctrine
was used only to adjudge the rights and liabilities of each parties in these kind of transactions.

DIGEST No. 2
FACTS:
Plaintiff is a corporation duly organized and existing under and by virtue of the laws of
the Philippines, with principal office in Manila, the capital stock of which is divided into 1,000
shares of P100 each. The Koppel Industrial Car and Equipment company, a corporation
organized and existing under the laws of the State of Pennsylvania, United States of America,
and not licensed to do business in the Philippines, owned nine hundred and 995 shares out of
the total capital stock of the plaintiff. The remaining 5 shares only were and are owned one each
by officers of the plaintiff corporation. That plaintiff, at all times material to this case, was and
now is duly licensed to engage in business as a merchant and commercial broker in the
Philippines; and was and is the holder of the corresponding merchant's and commercial broker's
privilege tax receipts.

the final say even as to how much should be allotted to said local entity in the so-called sharing
in the profits. SC further ruled that, it cannot overlook the fact that in the practical working of
corporate organizations of the class to which these two entities belong, the holder or holders of
the controlling part of the capital stock of the corporation, particularly where the control is
determined by the virtual ownership of the totality of the shares, dominate not only the selection
of the Board of Directors but, more often than not, also the action of that Board. Philippine
corporation could not possibly contravene with the American corporation in this case under
Exhibit H. This fact necessarily leads to the inference that the corporation had at least a VicePresident, and presumably also a President, who were not resident in the Philippines but in
America, where the parent corporation is domiciled. If Koppel (Philippines), Inc., had been
intended to operate as a regular domestic corporation in the Philippines, where it was formed,
the record and the evidence do not disclose any reason why all its officers should not reside and
perform their functions in the Philippines.

Exhibited H of the evidence: It is clearly understood that the intent of this contract is
that the broker shall perform only the functions of a broker as set forth above, and shall not take
possession of any of the materials or equipment applying to said orders or perform any acts or
duties outside the scope of a broker; and in no sense shall this contract be construed as
granting to the broker the power to represent the principal as its agent or to make commitments
on its behalf. The Court of First Instance held for the defendant and dismissed plaintiff's
complaint with costs to it.
(23) Philippine National Bank vs. Ritratto Group, Inc.
G.R. No. 142616
July 31, 2001
Lessons Applicable: Dealings with Corp. and Stockholders (Corporate Law)
ISSUE: Whether or not Koppel Philippines is a domestic corporation distinct and separate from,
and not a mere branch of Koppel Industrial Car and Equipment Co
RULING:
Koppel Philippines is a mere branch, subsidiary or agency of the latter. A corporation
will be looked upon as a legal entity as a general rule, and until sufficient reason to the contrary
appears; but, when the notion of legal entity is used to defeat public convenience, justify wrong,
protect fraud, or defend crime, the law will regard the corporation as an association of persons.
The corporate entity is disregarded where it is so organized and controlled, and its affairs are so
conducted, as to make it merely an instrumentality, agency, conduit or adjunct of another
corporation.
SC reasoned that, in so far as the sales involved herein are concerned, Koppel
Philippines, Inc., and Koppel Industrial Car and Equipment company are to all intents and
purposes one and the same; or, to use another mode of expression, that, as regards those
transactions, the former corporation is a mere branch, subsidiary or agency of the latter. This is
conclusively borne out by the fact, among others, that the amount of the so-called "share in the
profits" of Koppel (Philippines), Inc., was ultimately left to the sole, unbridled control of Koppel
Industrial Car and Equipment Company. No group of businessmen could be expected to
organize a mercantile corporation the ultimate end of which could only be profit if the
amount of that profit were to be subjected to such a unilateral control of another corporation,
unless indeed the former has previously been designed by the incorporators to serve as a mere
subsidiary, branch or agency of the latter. Evidently, Koppel Industrial Car and Equipment
Company made us of its ownership of the overwhelming majority 99.5% of the capital
stock of the local corporation to control the operations of the latter to such an extent that it had

FACTS:

May 29, 1996: PNB International Finance Ltd. (PNB-IFL) a subsidiary company of
PNB, organized and doing business in Hong Kong, extended a letter of credit in favor of
the Ritratto Group, Inc. (Ritartto) in the amount of US$300K secured by real estate
mortgages constituted over 4 parcels of land in Makati City

September 1996: increased successively to US$1,140,000.00

November 1996: to US$1,290,000.00

February 1997: US$1,425,000.00

April 1998: decreased to US$1,421,316.18

Ritratto Group, Inc. made repayments of the loan incurred by remitting those amounts
to their loan account with PNB-IFL in Hong Kong.
April 30, 1998: outstanding amounted to US$1,497,274.70

PNB-IFL, through its attorney-in-fact PNB, notified them of the foreclosure of


all the real estate mortgages and that the properties subjected

May 25, 1999: Ritratto Group, Inc filed a complaint for injunction with prayer for the

issuance of a writ of preliminary injunction and/or temporary restraining order before the
RTC. -granted 72-hour TRO

RTC and CA: dismissed motion to dismiss

PNB-IFL, is a wholly owned subsidiary of defendant Philippine National


Bank, the suit against the defendant PNB is a suit against PNB-IFL

Rittratto: entire credit facility is void as it contains stipulations in


violation of the principle of mutuality of contracts

ISSUE: W/N PNB is an alter ego of PNB-IFL

HELD: NO. Petition is granted

PNB is an agent with limited authority and specific duties under a special power of attorney
incorporated in the real estate mortgage.

Not privy to the loan contracts entered into by PNB-IFL.

Mere fact that a corporation owns all of the stocks of another corporation, taken alone
is not sufficient to justify their being treated as one entity.

If used to perform legitimate functions, a subsidiary's separate existence may be


respected, and the liability of the parent corporation as well as the subsidiary will be
confined to those arising in their respective business.

General rule the stock ownership alone by one corporation of the stock of another
does not thereby render the dominant corporation liable for the torts of the subsidiary
unless the separate corporate existence of the subsidiary is a mere sham, or unless the
control of the subsidiary is such that it is but an instrumentality or adjunct of the dominant
corporation.
The Circumstance
circumstances)

rendering

the

subsidiary

an

instrumentality (common

(a) The parent corporation owns all or most of the capital stock of the subsidiary.
(b) The parent and subsidiary corporations have common directors or officers.
(c) The parent corporation finances the subsidiary.
(d) The parent corporation subscribes to all the capital stock of the subsidiary or otherwise
causes its incorporation.
(e) The subsidiary has grossly inadequate capital.
(f) The parent corporation pays the salaries and other expenses or losses of the subsidiary.
(g) The subsidiary has substantially no business except with the parent corporation or no assets
except those conveyed to or by the parent corporation.
(h) In the papers of the parent corporation or in the statements of its officers, the subsidiary is
described as a department or division of the parent corporation, or its business or financial
responsibility is referred to as the parent corporation's own.
(i) The parent corporation uses the property of the subsidiary as its own.
(j) The directors or executives of the subsidiary do not act independently in the interest of the
subsidiary but take their orders from the parent corporation.
(k) The formal legal requirements of the subsidiary are not observed.

(24) KUKAN INTERNATIONAL CORPORATION VS. HON. AMOR REYES


G.R. NO. 182729, SEPTEMBER 29, 2010
FACTS:
Private respondent Romeo M. Morales doing business under the name RM Morales
Trophies and Plaques was awarded a P5 million contract for the supply and installation of
signages in a building constructed in Makati sometime in March 1998. The contract price was
later reduced to P3,388,502 because some items were deleted from the contract. Morales
complied with his contractual obligations but he was paid only the amount of P1,976,371.07
leaving a balance of P1,412,130.93. He filed a case against Kukan, Inc., for sum of money with
the RTC of Manila docketed as Civil Case No. 99-93173. Kukan Inc., stopped participating in the
proceedings in November 2000, hence, it was declared in default and Morales presented his
evidence ex-parte against petitioner.
On November 28, 2002, the RTC rendered a decision in favor of Morales and against
Kunkan, Inc. ordering the latter to pay the sum of P1,201,724.00 with legal interest of 12% per
annum until fully paid; P50,000.00 as moral damages,P20,000.00 as attorney's fees and
P7,960.06 as litigation expenses. The counterclaimfiled by Kunkan, Inc. was dismissed. The
decision became final and executory During the execution, the sheriff levied the personal
properties found at the office of Kukan, Inc.. Claiming it owned the properties levied, Kukan
International Corporation (KIC) fied an Affidavit of Third Party Claim. Morales filed an Omnibus
Motion praying to apply the principle of piercing the veil of corporate entity. He alleged that
Kankun, Inc. and KIC are one and the same corporation His Motion was denied. On Motion of
Morales the presiding Judge of Branch 17 of RTC Manila inhibited himself from hearing the
case. It was raffled to Branch 21 which granted the Motion filed by Morales on March 12, 2007
and decreed that Kukan, Inc. and Kukan International Inc., as one and the same corporation;
that the levy made on the properties of KIC is valid; and ordering Kunkan International Corp. and
Michael Chan as jointly and severally liable to pay the award pursuant to the Decision dated

November 28, 2002. KIC filed a Motion for Reconsideration which was denied.KIC brought the
case to the Court of Appeals which rendered the Decision n January 23, 2008 denying KIC's
petition. The CA also denied its Motion for Reconsideration in the Resolution dated June 7,
2007.

Petition granted.

Hence, this case.


ISSUE/S: One of the issues raised is whether or not the trial court and the appellate court
correctly applied the principle of piercing the veil of corporate entity.
HELD: The Supreme Court ruled that the doctrine of piercing the veil of corporate entity finds
no application in this case.
According to the Supreme Court, the principle of piercing the veil of corporate entity
and the resulting treatment of two related corporation as one and the same juridical person
applies only to established liability and not to confer jurisdiction. In this case, the Supreme Court
ruled that KIC was not made a party defendant in Civil Case No. 99-93173. It entered a special
but not a voluntary appearance in the trial court to assert that it was a separate entity and has a
separate legal personality from Kunkan, Inc. KIC was not impleaded nor served with summons.
Hence, it could only assert its claim through the affidavits, comments and motions filed by
special apperance before the RTC that it is a separate juridical entity.
The Supreme stated that the doctrine of piercing the veil of corporate entity comes to
play during the trial of the case after the court has already acquired jurisdiction over the
corporation.
To justify the piercing of the veil of corporate fiction, it must be shown by clear and
convincing proof that the separate and distinct personality of the corporation was purposely
employed to evade a legitimate and binding comittment and perpetuate a fraud or like a
wrongdoings.

(25) FIRST PHILIPPINE INTERNATIONAL BANK VS CA (252 SCRA 259)

In those instances when the Court pierced the veil of corporate fiction of two
corporations, there was a confluence of the following factors:
1.

A first corporation is dissolved;

2.
The assets of the first corporation is transferred to a second corporation to avoid a
financial liability of the first corporation; and
3.
Both corporations are owned and controlled by the same persons such that the second
corporation should be considered as a continuation and successor of the first corporation.
In this case, the second and third factors are conspicuously absent. There is,
therefore, no compelling justification for disregarding the fiction of corporate entity separating
Kukan, Inc. from KIC. In applying the principle, both the RTC and the CA miserably failed to
identify the presence of the abovementioned factors.
The High Court stated that neither should the level of paid-up capital of Kukan, Inc.
upon its incorporation be viewed as a badge of fraud, for it is in compliance with Sec. 13 of the
Corporation Code, which only requires a minimum paid-up capital of PhP 5,000.
The suggestion that KIC is but a continuation and successor of Kukan, Inc., owned
and controlled as they are by the same stockholders, stands without factual basis. The fact that
Michael Chan, a.k.a. Chan Kai Kit, owns 40% of the outstanding capital stock of both
corporations standing alone, is insufficient to establish identity. There must be at least a
substantial identity of stockholders for both corporations in order to consider this factor to be
constitutive of corporate identity.

First Philippine International Bank vs Court of Appeals


252 SCRA 259 [GR No. 115849 January 24, 1996]

Facts: In the course of its banking operations, the defendant Producer Bank of the Philippines
acquired 6 parcels of land with a total area of 101 hectares located at Don Jose, Sta. Rosa,
Laguna and covered by TCT No. T-106932 to T-106937. The property used to be owned by
BYME Investment and Development Corporation which hd them mortgaged with the bank as
collateral for a loan. The plaintiff originals, Demetrio Demetria and Jose Janolo wanted to
purchase the property and thus initiated negotiations for that purpose. In the early part of August
1987 said plaintiffs, upon the suggestion of BYME investments legal counsel, Fajardo met with
defendant Mercurio Rivera, manager of the property management department of the defendant
bank. The meeting was held in pursuant to plaintiffs plan to buy the property. After the meeting,
plaintiff Janolo, following the advice of defendant Rivera made a formal purchase offer to the
Bank through a letter dated August 30,1987. Negotiations took place and an offer price was fixed
at P5.5million. During the course of the negotiations, the defendant bank was placed under
conservatorship and a new conservator was appointed to which the name has been refused to
recognize. A derivative suit has been filed against Rivera for the damages suffered from the
alleged perfect contract of sale involving the 6 parcels of land.

Issue: Whether or not a derivative suit may lie involving the bank and its stockholders.

some other person, for his own ultimate benefit.

Held: No. An individual stockholder is permitted to institute a derivative suit on behalf of the
corporation wherein he hold stock in order to protect or vindicate corporate rights, whenever the
officials of the corporation refuse to sue, or are the ones, to be sued or hold the control of the
corporation. In such actions, the suing stockholder is regarded as a nominal party with the
corporation as the real party in interest.

Section 28-A of BP 68 merely gives the conservator power to revoke contracts that are, under
existing law, deemed not to be effective i.e void, voidable, unenforceable or rescissible. Hence,
the conservator merely takes the place of a banks board of directors. What the said board
cannot do such as repudiating a contract validly entered into under the doctrine of implied
authority the conservator cannot do either.

In the face of the damaging admissions taken from the complaint in the second case, petitioners,
quite strangely, sought to deny that the second case was a derivative suit, reasoning that it was
brought not by the minority shareholders, but by Henry Co. etal. who not only hold or control
over 80% of the outstanding capital stock, but also constitute the majority in the board of
directors of petitioners bank. That being so, then they really represent the bank, so whether they
sued derivatively or directly, there is undeniably an identity of interest/entity represented.

In addition to the many cases, where the corporate fiction has been regarded, we now add the
instant case, and declare herewith that the corporate veil cannot be used to shield an otherwise
blatant violation of the prohibition against forum shopping. Shareholders, whether suing as the
majority in direct actions or as the minority in a derivative suit, cannot be allowed to trifle with
court processes particularly where, as in this case, the corporation itself has not been remiss in
vigorously prosecuting or defending corporate causes and in using and applying remedies
available to it. To rule otherwise would be to encourage corporate litigants to use their
shareholders as fronts to circumvent the stringent rules against forum shopping.

From the facts, the official bank price, at any rte, the bank placed its official, Rivera is a position
of authority to accept offers to buy and negotiate the sale by having the offer officially acted upon
by the bank. The bank cannot turn around and say, as it now does, that what Rivera states as
the banks action on the matter is not in fact so. It is a familiar doctrine, the doctrine of ostensible
authority, that if a corporation on knowingly permits one of its officers, or any other agent, to do
acts within the scope of apparent authority, and thus holds him out to the public as possessing
power to do those acts, the corporation will, as against anyone who has in good faith dealt with
the corporation through such agent, he estopped from denying his authority.

(25) FIRST PHILIPPINE INTERNATIONAL BANK


vs.
COURT OF APPEALS
G.R. No. 115849. January 24, 1996
FACTS:
In the course of its banking operations, the defendant Producer Bank of the
Philippines acquired six parcels of land. The original plaintiffs, Demetrio Demetria and Jose O.
Janolo, wanted to purchase the property and thus initiated negotiations for that purpose.
Negotiations happened between the parties. However, petitioner bank reneged their
agreement because it offered the same lot to different buyers. Plaintiffs then filed a suit for
specific performance with damages against the bank, its Manager Rivers and Acting
Conservator Encarnacion. The basis of the suit was that the transaction had with the bank
resulted in a perfected contract of sale.
Subsequently, Henry L. Co, filed a motion to intervene in the trial court, alleging that
as owner of 80% of the Bank's outstanding shares of stock, he had a substantial interest in
resisting the complaint. The trial court issued an order denying the motion to intervene on the
ground that it was filed after trial had already been concluded. Henry Co did not appeal the
denial of his motion for intervention.
During the pendency of the proceedings in the Court of Appeals, Henry Co and
several other stockholders of the Bank, filed an action purportedly a "derivative suit" with the
RTC Branch 134, against Encarnacion, Demetria and Janolo to declare any perfected sale of the
property as unenforceable and to stop Ejercito from enforcing or implementing the sale. In his
answer, Janolo argued that the Second Case was barred by litis pendentia by virtue of the case
then pending in the Court of Appeals.
ISSUE:
Whether or not the juridical personalities of the two corporations be pierced.
RULING:
YES.

A bank is liable for wrongful acts of its officers done in the interest of the bank or in the course of
dealings of the officers in their representative capacity but not for acts outside the scope of their
authority. A bank holding out its officers and agents as worthy of confidence will not be permitted
to profit by the frauds they my thus be enabled to perpetrate in the apparent scope of their
employment; nor will it be permitted to shrink its responsibility for such fraud even through no
benefit may accrue to the bank therefrom. Accordingly, a banking corporation is liable to
innocent third persons where the representation is made in the course of its business by an
agent acting within the general scope of its authority even though, in the particular case, the
agent is secretly abusing his authority and attempting to perpetrate fraud upon his principal or

In addition to the many cases where the corporate fiction has been disregarded, we
now add the instant case, and declare herewith that the corporate veil cannot be used to shield
an otherwise blatant violation of the prohibition against forum-shopping. Shareholders, whether
suing as the majority in direct actions or as the minority in a derivative suit, cannot be allowed to
trifle with court processes, particularly where, as in this case, the corporation itself has not been
remiss in vigorously prosecuting or defending corporate causes and in using and applying
remedies available to it. To rule otherwise would be to encourage corporate litigants to use their
shareholders as fronts to circumvent the stringent rules against forum shopping.

(26) G.R. No. L-35262

March 15, 1930

THE PEOPLE OF THE PHILIPPINE ISLANDS, plaintiff-appellant,


vs.
TAN BOON KONG, defendant-appellee.
OSTRAND, J.:
This is an appeal from an order of the Judge of the Twenty-third Judicial District sustaining to
demurrer to an information charging the defendant Tan Boon Kong with the violation of section
1458 of Act No. 2711 as amended. The information reads as follows:
That on and during the four quarters of the year 1924, in the municipality of Iloilo,
Province of Iloilo, Philippine Islands, the said accused, as corporation organized under
the laws of the Philippine Islands and engaged in the purchase and the sale of sugar,
"bayon," coprax, and other native products and as such object to the payment of
internal-revenue taxes upon its sales, did then and there voluntarily, illegally, and
criminally declare in 1924 for the purpose of taxation only the sum of P2,352,761.94,
when in truth and in fact, and the accused well knew that the total gross sales of said
corporation during that year amounted to P2543,303.44, thereby failing to declare for
the purpose of taxation the amount of P190,541.50, and voluntarily and illegally not
paying the Government as internal-revenue percentage taxes the sum of P2,960.12,
corresponding to 1 per cent of said undeclared sales.

cited.)
In case of State vs. Burnam (17 Wash., 199), the court went so far as to hold that the manager
of a diary corporation was criminally liable for the violation of a statute by the corporation
through he was not present when the offense was committed.
In the present case the information or complaint alleges that he defendant was the manager of a
corporation which was engaged in business as a merchant, and as such manager, he made a
false return, for purposes of taxation, of the total amount of sale made by said false return
constitutes a violation of law, the defendant, as the author of the illegal act, must necessarily
answer for its consequences, provided that the allegation are proven.
The ruling of the court below sustaining the demurrer to the complaint is therefore reversed, and
the case will be returned to said court for further proceedings not inconsistent with our view as
hereinafter stated. Without costs. So ordered.

Lessons Applicable: Corporate Criminal Liability (Corporate Law)


FACTS:

The question to be decided is whether the information sets forth facts rendering the defendant,
as manager of the corporation liable criminally under section 2723 of Act No. 2711 for violation of
section 1458 of the same act for the benefit of said corporation. Section 1458 and 2723 read as
follows:

Tan Boon Kong, manager of the Visayan Gen. Supply Co. Inc, enegaed in the
purchase and sale of sugar "bayon:, copra and other native projects voluntarily made a
false return stating gross sales of only 2,352,761.94 when the true amount is 2,543,303. 44
with a difference of 190,541.50 (1 1/2 sales) resulting to a tax difference of 2,960.12.

SEC. 1458. Payment of percentage taxes Quarterly reports of earnings. The


percentage taxes on business shall be payable at the end of each calendar quarter in
the amount lawfully due on the business transacted during each quarter; and it shall
be on the duty of every person conducting a business subject to such tax, within the
same period as is allowed for the payment of the quarterly installments of the fixed
taxes without penalty, to make a true and complete return of the amount of the
receipts or earnings of his business during the preceeding quarter and pay the tax due
thereon. . . . (Act No. 2711.)
SEC. 2723. Failure to make true return of receipts and sales. Any person who,
being required by law to make a return of the amount of his receipts, sales, or
business, shall fail or neglect to make such return within the time required, shall be
punished by a fine not exceeding two thousand pesos or by imprisonment for a term
not exceeding one year, or both.
And any such person who shall make a false or fraudulent return shall be punished by
a fine not exceeding ten thousand pesos or by imprisonment for a term not exceeding
two years, or both. (Act No. 2711.)
Apparently, the court below based the appealed ruling on the ground that the offense charged
must be regarded as committed by the corporation and not by its officials or agents. This view is
in direct conflict with the great weight of authority. a corporation can act only through its officers
and agent s, and where the business itself involves a violation of the law, the correct rule is that
all who participate in it are liable (Grall and Ostrand's Case, 103 Va., 855, and authorities there

Secs. 1458 to 2723 seem to mention only about corporations

ISSUE: W/N Tan Boon Kong is criminally liable.


HELD: YES.
A corporation can act only through its officers and agents and where the

business itself involves a violation of the law, the correct rule is that all who participate in it
= liable

(27) PNB vs CA
G.R. No. L-27155 May 18, 1978
Facts:
Defendant Rita Guenco Tapnio secured a crop loan from PNB. This crop loan was secured by a
mortgage on her standing crop including her sugar quota allocation for the agricultural year
corresponding to said standing crop. Philmagen executed its Bond, with defendant Rita Gueco
Tapnio as principal, in favor of the Philippine National Bank Branch at San Fernando,
Pampanga, to guarantee the payment of defendant Rita Gueco Tapnio's account with said Bank.
In turn, to guarantee the payment of whatever amount the bonding company would pay to the
Philippine National Bank, both defendants (Rita Gueco Tapnio and Cecilio Gueco) executed the
indemnity agreement.
It is not disputed that defendant Rita Gueco Tapnio was indebted to the bank in the sum of
P2,000.00, plus accumulated interests unpaid, which she failed to pay despite demands. The
Bank wrote a letter of demand to Philmagen, whereupon Philmagen accordingly paid the full
amount due and owing in the sum of P2,379.91, for and on account of defendant Rita Gueco's
obligation.
Defendant Rita Gueco Tapnio admitted all the foregoing facts. She claims, however, when
demand was made upon her by plaintiff for her to pay her debt to the Bank, that she told the
Plaintiff that she did not consider herself to be indebted to the Bank at all because she had an
agreement with one Jacobo-Nazon whereby she had leased to the latter her unused export
sugar quota for the 1956-1957 agricultural year. his lease agreement, according to her, was with
the knowledge of the bank. But the Bank has placed obstacles to the consummation of the
lease, and the delay caused by said obstacles forced 'Nazon to rescind the lease
contract. Thus, Rita Gueco Tapnio filed her third-party complaint against the Bank to recover
from the latter any and all sums of money which may be adjudged against her and in favor of the
plaitiff plus moral damages, attorney's fees and costs.
Sometimes, a planter harvest less sugar than her quota, so her excess quota is utilized by
another who pays her for its use. This is the arrangement entered into between Mrs. Tapnio and
Mr. Tuazon regarding the former's excess quota for 1956-1957.
Since the quota was mortgaged to the P.N.B., the contract of lease had to be approved by said
Bank, The same was submitted to the branch manager at San Fernando, Pampanga.
Consideration of the evidence discloses that when the branch manager of the Philippine
National Bank at San Fernando recommended the approval of the contract of lease at the price
of P2.80 per picul, whose recommendation was concurred in by the Vice-president of said Bank,
J. V. Buenaventura, the board of directors required that the amount be raised to 13.00 per picul.
Mr. Tuazon asked for a reconsideration of the price per picul but thaw same was not acted upon
the PNBs BOD. The parties were notified of the refusal on the part of the board of directors of
the Bank to grant the motion for reconsideration. As such, Tuazon wrote a letter to the Bank
informing the Bank that he was no longer interested to continue the deal, referring to the lease of
sugar quota allotment in favor of defendant Rita Gueco Tapnio. The result is that the latter lost
the sum of P2,800.00 which she should have received from Tuazon and which she could have
paid the Bank to cancel off her indebtedness.
Issue/Held: WON the rescission of the lease contract of the 1,000 piculs of sugar quota
allocation of respondent Rita Gueco Tapnio by Jacobo C. Tuazon was due to the
unjustified refusal of petitioner to approve said lease contract, and its unreasonable
insistence on the rental price of P3.00 instead of P2.80 per picul.- YES
Ratio: It has been clearly shown that when the Branch Manager of petitioner required the

parties to raise the consideration of the lease from P2.50 to P2.80 per picul, or a total of P2,80000, they readily agreed. Hence, in his letter to the Branch Manager of the Bank on August 10,
1956, Tuazon informed him that the minimum lease rental of P2.80 per picul was acceptable to
him and that he even offered to use the loan secured by him from petitioner to pay in full the sum
of P2,800.00 which was the total consideration of the lease. This arrangement was not only
satisfactory to the Branch Manager but it was also approves by Vice-President J. V.
Buenaventura of the PNB. Under that arrangement, Rita Gueco Tapnio could have realized the
amount of P2,800.00, which was more than enough to pay the balance of her indebtedness to
the Bank which was secured by the bond of Philamgen.
There is no question that Tapnio's failure to utilize her sugar quota for the crop year 1956-1957
was due to the disapproval of the lease by the Board of Directors of petitioner.
Time is of the essence in the approval of the lease of sugar quota allotments, since the same
must be utilized during the milling season, because any allotment which is not filled during such
milling season may be reallocated by the Sugar Quota Administration to other holders of
allotments. There was no proof that there was any other person at that time willing to lease the
sugar quota allotment of private respondents for a price higher than P2.80 per picul. "The fact
that there were isolated transactions wherein the consideration for the lease was P3.00 a picul",
according to the trial court, "does not necessarily mean that there are always ready takers of
said price." The unreasonableness of the position adopted by the petitioner's Board of Directors
is shown by the fact that the difference between the amount of P2.80 per picul offered by Tuazon
and the P3.00 per picul demanded by the Board amounted only to a total sum of P200.00.
Issue/Held: WON PNB is liable for the damage caused.- YES
Ratio: While petitioner had the ultimate authority of approving or disapproving the proposed
lease since the quota was mortgaged to the Bank, the latter certainly cannot escape its
responsibility of observing, for the protection of the interest of private respondents, that degree
of care, precaution and vigilance which the circumstances justly demand in approving or
disapproving the lease of said sugar quota.
The law makes it imperative that every person "must in the exercise of his rights and in the
performance of his duties, act with justice, give everyone his due, and observe honesty and
good faith. This petitioner failed to do. Certainly, it knew that the agricultural year was about to
expire, that by its disapproval of the lease private respondents would be unable to utilize the
sugar quota in question.
In failing to observe the reasonable degree of care and vigilance which the surrounding
circumstances reasonably impose; petitioner is consequently liable for the damages caused on
private respondents. Under Article 21 of the New Civil Code, "any person who wilfully causes
loss or injury to another in a manner that is contrary to morals, good customs or public policy
shall compensate the latter for the damage." The afore-cited provisions on human relations were
intended to expand the concept of torts in this jurisdiction by granting adequate legal remedy for
the untold number of moral wrongs which is impossible for human foresight to specifically
provide in the statutes.
A corporation is civilly liable in the same manner as natural persons for torts, because
"generally speaking, the rules governing the liability of a principal or master for a tort
committed by an agent or servant are the same whether the principal or master be a
natural person or a corporation, and whether the servant or agent be a natural or artificial
person. All of the authorities agree that a principal or master is liable for every tort which
he expressly directs or authorizes, and this is just as true of a corporation as of a natural
person, A corporation is liable, therefore, whenever a tortious act is committed by an
officer or agent under express direction or authority from the stockholders or members
acting as a body, or, generally, from the directors as the governing body."

PNB vs. CA

83 SCRA 237 Business Organization Corporation Law Corporations Liability for


Negligence
FACTS: Rita Tapnio owes PNB an amount of P2,000.00. The amount is secured by her sugar
crops about to be harvested including her export quota allocation worth 1,000 piculs. The said
export quota was later dealt by Tapnio to a certain Jacobo Tuazon at P2.50 per picul or a total of
P2,500. Since the subject of the deal is mortgaged with PNB, the latter has to approve it. The
branch manager of PNB recommended that the price should be at P2.80 per picul which was the
prevailing minimum amount allowable. Tapnio and Tuazon agreed to the said amount. And so
the bank manager recommended the agreement to the vice president of PNB. The vice
president in turn recommended it to the board of directors of PNB.

However, the Board of Directors wanted to raise the price to P3.00 per picul. This Tuazon does
not want hence he backed out from the agreement. This resulted to Tapnio not being able to
realize profit and at the same time rendered her unable to pay her P2,000.00 crop loan which
would have been covered by her agreement with Tuazon.

Eventually, Tapnio was sued by her other creditors and Tapnio filed a third party complaint
against PNB where she alleged that her failure to pay her debts was because of PNBs
negligence and unreasonableness.
ISSUE: Whether or not Tapnio is correct.

HELD: Yes. In this type of transaction, time is of the essence considering that Tapnios sugar
quota for said year needs to be utilized ASAP otherwise her allotment may be assigned to
someone else, and if she cant use it, she wont be able to export her crops. It is unreasonable
for PNBs board of directors to disallow the agreement between Tapnio and Tuazon because of
the mere difference of 0.20 in the agreed price rate. What makes it more unreasonable is the
fact that the P2.80 was recommended both by the bank manager and PNBs VP yet it was
disapproved by the board. Further, the P2.80 per picul rate is the minimum allowable rate
pursuant to prevailing market trends that time. This unreasonable stand reflects PNBs lack of
the reasonable degree of care and vigilance in attending to the matter. PNB is therefore
negligent.

A corporation is civilly liable in the same manner as natural persons for torts, because generally
speaking, the rules governing the liability of a principal or master for a tort committed by an
agent or servant are the same whether the principal or master be a natural person or a

corporation, and whether the servant or agent be a natural or artificial person. All of the
authorities agree that a principal or master is liable for every tort which it expressly directs or
authorizes, and this is just as true of a corporation as of a natural person, a corporation is liable,
therefore, whenever a tortious act is committed by an officer or agent under express direction or
authority from the stockholders or members acting as a body, or, generally, from the directors as
the governing body.

Restraining Order and/ or Writ of Preliminary Injunction.


The spouses claimed that the foreclosure of the real estate mortgages is illegal because BPI
should have exhausted CCCCs properties first, stressing that they are mere guarantors of the
renewed loans. They also prayed that they be awarded moral and exemplary damages,
attorneys fees, litigation expenses and cost of suit.

(28) HERMAN C. CRYSTAL vs. BANK OF THE PHILIPPINE ISLANDS


G.R. No. 172428 November 28, 2008
Facts:
On 28 March 1978, spouses Raymundo and Desamparados Crystal obtained a P300,000.00
loan in behalf of the Cebu Contractors Consortium Co. (CCCC) from the BPI-Butuan. The loan
was secured by a chattel mortgage on heavy equipment and machinery of CCCC. Thereafter, or
on 29 March 1979, Raymundo Crystal executed a promissory note for the amount of
P300,000.00, also in favor of BPI-Butuan.
Sometime in August 1979, CCCC renewed a previous loan, this time from BPI, Cebu City
branch. The renewal was evidenced by a promissory note dated 13 August 1979, signed by the
spouses in their personal capacities and as managing partners of CCCC.

The trial court dismissed the spouses complaint and ordered them to pay moral and exemplary
damages and attorneys fees to BPI. It ruled that since the spouses agreed to bind themselves
jointly and severally, they are solidarily liable for the loans; hence, BPI can validly foreclose the
two real estate mortgages. Moreover, being guarantors-mortgagors, the spouses are not entitled
to the benefit of exhaustion.
The spouses appealed the decision of the trial court to the Court of Appeals, but their appeal
was dismissed. The spouses moved for the reconsideration of the decision, but the Court of
Appeals also denied their motion for reconsideration.
Issue:
1)
2)
3)

Whether or not the liability is extinguished;


Whether or not Spouses are solidarily liable with the corporations debt; and
Whether or not they are entitled to moral damages

Held:
The contention has no merit.

The promissory note states that the spouses are jointly and severally liable with CCCC. It
appears that before the original loan could be granted, BPI-Cebu City required CCCC to put up
a security. However, CCCC had no real property to offer as security for the loan; hence, the
spouses executed a real estate mortgage over their own real property on 22 September 1977.

1.

On 3 October 1977, they executed another real estate mortgage over the same lot in favor of
BPI-Cebu City, to secure an additional loan of P20,000.00 of CCCC.
CCCC failed to pay its loans to both BPI-Butuan and BPI-Cebu City when they became due
despite demands.
Thus, BPI resorted to the foreclosure of the chattel mortgage and the real estate mortgage. The
foreclosure sale on the chattel mortgage was initially stalled with the issuance of a restraining
order against BPI.

Besides, under Art. 1236 of the Civil Code, the creditor is not bound to accept payment or
performance by a third person who has no interest in the fulfillment of the obligation, unless
there is a stipulation to the contrary. We see no stipulation in the promissory note which states
that a third person may fulfill the spouses obligation. Thus, it is clear that the spouses alone
bear responsibility for the same.
2.

However, following BPIs compliance with the necessary requisites of extrajudicial foreclosure,
the foreclosure sale on the chattel mortgage was consummated on 28 February 1988, with the
proceeds amounting to P240,000.00 applied to the loan from BPI-Butuan which had then
reached P707,393.90.
Meanwhile, on 7 July 1981, Insular Bank of Asia and America (IBAA), through its Vice-President
for Legal and Corporate Affairs, offered to buy the lot subject of the two (2) real estate
mortgages and to pay directly the spouses indebtedness in exchange for the release of the
mortgages. BPI rejected IBAAs offer to pay.
BPI filed a complaint for sum of money against CCCC and the spouses before the Regional Trial
Court of Butuan City, seeking to recover the deficiency of the loan of CCCC and the spouses
with BPI-Butuan.
The trial court ruled in favor of BPI. Pursuant to the decision, BPI instituted extrajudicial
foreclosure of the spouses mortgaged property.
On 10 April 1985, the spouses filed an action for Injunction With Damages, With A Prayer For A

Petitioners rely on IBAAs offer to purchase the mortgaged lot from them and to
directly pay BPI out of the proceeds thereof to settle the loan. BPIs refusal to agree to
such payment scheme cannot extinguish the spouses loan obligation. In the first
place, IBAA is not privy to the loan agreement or the promissory note between the
spouses and BPI. Contracts, after all, take effect only between the parties, their
successors in interest, heirs and assigns.

A solidary obligation is one in which each of the debtors is liable for the entire
obligation, and each of the creditors is entitled to demand the satisfaction of the whole
obligation from any or all of the debtors. A liability is solidary "only when the obligation
expressly so states, when the law so provides or when the nature of the obligation so
requires."

Thus, when the obligor undertakes to be "jointly and severally" liable, it means that the obligation
is solidary, such as in this case. By stating "I/we promise to pay, jointly and severally, to the
BANK OF THE PHILIPPINE ISLANDS," the spouses agreed to be sought out and be demanded
payment from, by BPI. BPI did demand payment from them, but they failed to comply with their
obligation, prompting BPIs valid resort to the foreclosure of the chattel mortgage and the real
estate mortgages.
Thus we held in one case that if solidary liability was instituted to "guarantee" a principal
obligation, the law deems the contract to be one of suretyship.26 And while a contract of a
surety is in essence secondary only to a valid principal obligation, the suretys liability to the
creditor or promisee of the principal is said to be direct, primary, and absolute; in other words,
the surety is directly and equally bound with the principal. T

held not to be a ground for an award of moral damages.


3.

Moral damages are meant to compensate the claimant for any physical suffering,
mental anguish, fright, serious anxiety, besmirched reputation, wounded feelings,
moral shock, social humiliation and similar injuries unjustly caused.

Such damages, to be recoverable, must be the proximate result of a wrongful act or omission
the factual basis for which is satisfactorily established by the aggrieved party. There being no
wrongful or unjust act on the part of BPI in demanding payment from them and in seeking the
foreclosure of the chattel and real estate mortgages, there is no lawful basis for award of
damages in favor of the spouses.
Neither is BPI entitled to moral damages. A juridical person is generally not entitled to moral
damages because, unlike a natural person, it cannot experience physical suffering or such
sentiments as wounded feelings, serious anxiety, mental anguish or moral shock.
Indeed, while the Court may allow the grant of moral damages to corporations, it is not
automatically granted; there must still be proof of the existence of the factual basis of the
damage and its causal relation to the defendants acts. This is so because moral damages,
though incapable of pecuniary estimation, are in the category of an award designed to
compensate the claimant for actual injury suffered and not to impose a penalty on the
wrongdoer.
The spouses complaint against BPI proved to be unfounded, but it does not automatically entitle
BPI to moral damages. Although the institution of a clearly unfounded civil suit can at times be a
legal justification for an award of attorney's fees, such filing, however, has almost invariably been

The rationale for the rule is that the law could not have meant to impose a penalty on the right to
litigate. Otherwise, moral damages must every time be awarded in favor of the prevailing
defendant against an unsuccessful plaintiff. BPI may have been inconvenienced by the suit, but
we do not see how it could have possibly suffered besmirched reputation on account of the
single suit alone. Hence, the award of moral damages should be deleted.
The awards of exemplary damages and attorneys fees, however, are proper. Exemplary
damages, on the other hand, are imposed by way of example or correction for the public good,
when the party to a contract acts in a wanton, fraudulent, oppressive or malevolent manner,
while attorneys fees are allowed when exemplary damages are awarded and when the party to
a suit is compelled to incur expenses to protect his interest.
The spouses instituted their complaint against BPI notwithstanding the fact that they were the
ones who failed to pay their obligations. Consequently, BPI was forced to litigate and defend its
interest. For these reasons, BPI is entitled to the awards of exemplary damages and attorneys
fees.

You might also like